Chapter 1 Quiz: Spring 2021 BIO-215-OL-A: Nutrition, Chapter 18 Quiz: Spring 2021 BIO-215-OL-A: Nutrition, Chapter 17 Quiz: Spring 2021 BIO-215-OL-A: Nutrition, Chapter 19 Quiz: Spring 2021 BIO-215-OL-A: Nutrition, Chapter Quiz 16: Spring 2021 BIO-21...

Lakukan tugas rumah & ujian kamu dengan baik sekarang menggunakan Quizwiz!

The school nurse is teaching nutrition to a group of kindergarten students. After presenting MyPlate to the students and discussing their food options, the nurse knows teaching has been effective when she hears which of the following comments? 1. "Half our plate should be fruits and vegetables." 2. "We should drink whole milk." 3. "We should eat a sweet roll instead of candy." 4. "We should eat twice the grains as protein."

"Half our plate should be fruits and vegetables."

The nurse is conducting client education for a 71-year-old woman who is a resident of a long-term care facility. The assessment indicates that she is at a high risk for malnutrition. Which of the following reasons concerns is the client likely to report with respect to her nutritional status? "I just don't have an appetite." "I chew quite well with my dentures." "I eat most of my meals with friends." "I take two different medicines every day."

"I just don't have an appetite."

A 79-year-old client has been admitted to a long-term care facility where a liberal eating pattern is offered. The client is concerned about his long history of high cholesterol and wants a diet that will promote heart health. Which of the following is the nurse's best response? "You are too old to worry about your cholesterol now." "Malnutrition is a greater threat than hypercholesterolemia." "The facility does not offer specialized, individual diet options." "If the cholesterol hasn't gone down yet, it never will."

"Malnutrition is a greater threat than hypercholesterolemia."

When asked by a client if she should be concerned about the amount of phosphorus in her diet, which of the following is the best response by the nurse? "Only if you do not eat enough fruit and vegetables." "No, phosphorus is pervasive in the food supply." "No, phosphorus is added to foods because there are few good natural sources." "Only if all of your food comes from a grocery store."

"No, phosphorus is pervasive in the food supply."

The nurse is providing nutrition education to a group of clients recently diagnosed with diabetes. The nurse recognizes that more discussion is necessary when one of the participants say which of the following? "It's important to note the serving size." "One cup of any canned food equals one carbohydrate exchange." "If sugar is the first ingredient, there may be too much sugar in this product." "Some products may count in more than one group."

"One cup of any canned food equals one carbohydrate exchange."

When providing teaching to a client about the importance of drinking the recommended amount of water daily, the client states, "I just really hate the taste of water." What is the nurse's best response? "Other types of beverages can be more beneficial to the body than water." "Other types of beverages provide no beneficial caloric intake to the body." "Other types of beverages may provide phytochemicals to the body." "Other types of beverages are complete sources of fluids for the body."

"Other types of beverages may provide phytochemicals to the body."

Sodium is a necessary nutrient and the major extracellular cation in the body. When providing client education regarding the importance of sodium in the body, which of the following is the most accurate statement the nurse can make? "Sodium helps the muscles remain stable." "Sodium strengthens immune function." "Sodium helps the transmission of impulses between brain cells." "Sodium helps the heart remain calm."

"Sodium helps the transmission of impulses between brain cells."

The parents of a 9-month-old infant ask how to prevent obesity in their child. What would be the best age-appropriate response to their question? "The baby should be fed 8 oz of formula per month of age." "Be persistent with foods the baby refuses to ensure adequate nutrient intake." "You should follow the recommendations from the MyPlate food plan." "The baby will decide when he is full and give cues."

"The baby will decide when he is full and give cues."

A client who is 23 weeks pregnant tells the nurse she is considering drinking herbal tea to help relax in the evenings. Which of the following would be the nurse's best response? "Research shows many herbs induce early labor." "The safety of herbs in pregnancy is not known." "Herbal teas are known to cause nutritional deficits in babies." "Herbs have been found to be teratogenic to the fetus."

"The safety of herbs in pregnancy is not known."

The nurse is caring for a 21-year-old female with burns that cover more than 20% of her total body surface area (TBSA). The client's parents ask why the client is being fed by a nasogastric tube (NG). Which of the following is the nurse's best response? "If your daughter were eating normally, she would get sicker because her intestines are not working right." "Your daughter's intestines are not working normally right now so we have to feed her through a tube." "Trying to eat normally would just wear your daughter out, so we feed her through a tube to conserve her energy." "We need to get a lot of calories and nutrients into your daughter to help her heal, and this is the best way to do that."

"We need to get a lot of calories and nutrients into your daughter to help her heal, and this is the best way to do that."

A nurse is seeing a client who is 23 weeks pregnant. The client asks, "Is it really that necessary to take a prenatal vitamin?" Which of the following is the nurse's best response? "Yes, because the RDA for iron increases with pregnancy." "Yes, especially if you live somewhere with lots of sun." "No, because your RDA for calcium does not increase with pregnancy." "Not as long as you 350 mg of calcium per day."

"Yes, because the RDA for iron increases with pregnancy."

A nurse is lecturing a class of nursing students about nutrition in children. The students are given the assignment to plan a heart-healthy diet for an 8-year-old. Which of the following would be the best breakfast? 2 fried eggs, 2 pieces of white toast with 3 tsp of butter, 2 pieces of bacon, and 8 oz of orange juice 1 package of toaster pastries and 1 glass of apple juice 1 cup of oatmeal, 8 oz of 2% milk, and 1 piece of wheat toast with 1 tsp of butter 2 pieces of French toast, 1/4 cup of maple syrup, and 6 oz of grape juice

1 cup of oatmeal, 8 oz of 2% milk, and 1 piece of wheat toast with 1 tsp of butter

The nurse is helping the client develop a meal plan based on 2000-calorie MyPlate plan. Which of the following menus shows the client understands the plan? 1. 2 cups oatmeal with 2 tbsp brown sugar, 1/2 cup raisins and 1 cup whole milk, 1 cup hot tea with 1 tsp honey 2. 2 cups Rice Krispies with 1 cup low-fat milk, 1 cup sliced strawberries with 1 tsp sugar, 1 slice whole wheat toast with 1 tsp butter and grape jelly, 1 cup coffee black 3. 1 cup raspberry flavored yogurt with 1 tsp flax seeds, 2 sliced toasted sourdough bread with 1 tsp canola margarine, 1 cup coffee with creamer 4. 1 cup plain yogurt with fresh strawberry slices, 1 hard-boiled egg, 1 cup herbal tea, 1 slice 8-grain toast

1 cup plain yogurt with fresh strawberry slices, 1 hard-boiled egg, 1 cup herbal tea, 1 slice 8-grain toast

The nurse is teaching a client about diet changes due to his recent cholecystectomy with emphasis on low-fat choices. Which of the following is an example of a good menu for the client? 1. 5 buffalo sauce-coated chicken wings, 1 cup mashed potatoes with 1 tsp butter and sour cream, 2 cups green salad with lettuce, carrots, tomatoes, onions, radishes, bell peppers, and 2 tbsp oil and vinegar dressing 2. 1 cup sweet potatoes, 1 cup steamed collard greens, 3 oz wild rice, 3 oz ham 3. 1 hamburger bun with sesame seeds, 4 oz hamburger patty fried, 1 slice each American cheese, onion, tomato, and lettuce, 1 tbsp ketchup and mayo 4. 1/2 cup sweet potatoes with marshmallows, 1 cup steamed broccoli, 1 cup cornbread dressing, 4 oz ham

1 cup sweet potatoes, 1 cup steamed collard greens, 3 oz wild rice, 3 oz ham

The client is learning how to plan his meal using the food list for diabetes to maintain a 2000-calorie diet. Which of the following are correct guidelines for his carbohydrate choices per meal in the 2000-calorie plan? Select all that apply. 1 serving of fruit 1 serving from the bread, cereal, rice, and pasta group 2 servings from milk group 3 servings of nonstarchy foods

1 serving of fruit 1 serving from the bread, cereal, rice, and pasta group 3 servings of nonstarchy foods

The nurse has been teaching a client with type 2 diabetes in menu planning using the carbohydrate counting method based on a 2000-calorie diet. The nurse asked the client to plan several dinner menus. Which of the following menus demonstrates the client's understanding of carbohydrate counting? 1 taco shell, 3 oz of taco meat; 1 1/2 cups combined chopped lettuce, tomato, onion; 1/3 cup of rice; and 1 cup of papaya 1 1/2 cups of chicken noodle soup (1/2 cup of chicken breast, 1 cup of egg noodles, chopped carrots, onions, celery, and broth), 1 oz of saltine crackers, 1 cup of hot tea with 1 teaspoon of honey 1 cup of spaghetti noodles, 4 meatballs, 1/2 cup of spaghetti sauce; coleslaw, 1 slice of buttered garlic bread, and 1 cup of milk 4 oz of broiled hamburger patty, 1 cup of mashed potatoes with butter, 1 cup of green beans, and 1 cup of low-fat vanilla ice cream

1 taco shell, 3 oz of taco meat; 1 1/2 cups combined chopped lettuce, tomato, onion; 1/3 cup of rice; and 1 cup of papaya

When providing education to a client who weighs 260 pounds, the nurse should tell the client weight loss at which of the following rates should be expected for the first 6 months? 1. 3 to 4 pounds a week 2. 1 to 2 pounds a week 3. 2 to 3 pounds a week 4. 0.5 pounds a week

1 to 2 pounds a week

When assessing the client's understanding of the role of beta-carotene in health, which of the following statements indicate the client has a correct understanding of the role of this nutrient? 1. "I include winter squash in several of my meals; it helps my eyesight." 2. "I cook with vegetable oils only; I want to keep my cholesterol low." 3. "I have whole eggs every morning; I need the protein." 4. "I drink orange juice each day; it helps my immune system."

1. "I include winter squash in several of my meals; it helps my eyesight."

The client is interested in breastfeeding her infant but has to return to work once her maternity leave is over. The client says she is concerned she will not be able to breastfeed her baby long enough for optimal health. How should the nurse respond? 1. "It is recommended that you give the baby only breast milk for first 6 months of life." 2. "It is recommended that you give the baby only breast milk for the first 3 months of life." 3. "Formula can provide the baby with the exact nutrients found in breast milk." 4. "It is recommended that you give the baby only breast milk for the first 4 months of life."

1. "It is recommended that you give the baby only breast milk for first 6 months of life."

The nurse is suggesting a 1400 cal/day plan for a client who wants to lose weight. In choosing the balanced diet, which of the following foods can the client intake daily? Select all that apply. 1. 5 oz grilled chicken breast 2. 7 oz lean steak 3. 2 bunches of leafy greens 4. 5 tsp olive oil 5. 3 cups milk 6. 6 oz hamburger with bun

1. 5 oz grilled chicken breast 3. 2 bunches of leafy greens 4. 5 tsp olive oil

It is important for good health that an individual consume enough of all nutrients to meet the needs of the body. If there is insufficient intake for the energy needs of the body, then the body is in what? 1. A negative calorie balance 2. A positive caloric balance 3. A negative energy balance 4. A positive energy balance

1. A negative calorie balance

The nurse is providing education to families about identifying signs of an eating disorder. The nurse should include which of the following physical symptoms when discussing anorexia? Select all that apply. 1. Anemia 2. Diarrhea 3. Amenorrhea 4. Hypotension 5. Hypervitaminosis D

1. Anemia 3. Amenorrhea 4. Hypotension

It has been proven that taking folate at the recommended amount may help prevent neural tube defects. At which point in the pregnancy should folate be taken to prevent this complication? 1. Before conception and early in pregnancy 2. Throughout pregnancy 3. During the second trimester of pregnancy 4. During the third trimester of pregnancy

1. Before conception and early in pregnancy

A client asks the nurse in the prenatal clinic when she should plan to be tested for possible diabetes during her pregnancy. Which of the following is a correct response? 1. Between 24 and 28 weeks of gestation 2. Between 28 and 32 weeks of gestation 3. Between 20 and 24 weeks of gestation 4. Between 16 and 20 weeks of gestation

1. Between 24 and 28 weeks of gestation

The nurse is preparing an educational session on the long-term benefits of breastfeeding. Which of the following teaching points should the nurse include? 1. Breastfeeding is associated with reducing the risk of postpartum depression. 2. Breastfeeding is associated with more predictable menstruation. 3. Breastfeeding is associated with decreased risk of metastatic cancers. 4. Breastfeeding is associated with a prevention of later life obesity.

1. Breastfeeding is associated with reducing the risk of postpartum depression.

The nurse is meeting with several pregnant clients to discuss breastfeeding. Which of the following statements about the benefits of breastfeeding for mothers and infants should the nurse include in the discussion? Select all that apply. 1. Breastfeeding may decrease the risk of food allergies in the infant. 2. Breastfeeding may be protective against breast cancer in the mother. 3. Breastfeeding can mobilize fat stores to help women lose weight, particularly in the lower body. 4. Breastfeeding provides reliable contraception. 5. Breastfeeding decreases bone density.

1. Breastfeeding may decrease the risk of food allergies in the infant. 2. Breastfeeding may be protective against breast cancer in the mother. 3. Breastfeeding can mobilize fat stores to help women lose weight, particularly in the lower body.

The nurse explains to a client who is considering bariatric surgery why an assessment of body mass index (BMI) is important. Which of the following reasons would be appropriate for the nurse to include in her explanation? 1. Calculating the BMI helps in determining the risk of health problems. 2. Knowing your BMI helps determine how people metabolize fat. 3. BMI can help determine a person's body level of ghrelin. 4. A high BMI is associated with leptin deficiency.

1. Calculating the BMI helps in determining the risk of health problems.

The nurse is caring for a client who was admitted for intractable nausea and vomiting that has now been controlled. The physician has ordered the diet to be advanced as tolerated. Of the following foods, what is the best choice for this client? 1. Dry toast 2. Peanut butter and crackers 3. Ice cream 4. Cream soup

1. Dry toast

1. The nurse is conducting nutrition counseling for a 21-year-old woman in the clinic area. The nurse is discussing calories for others uses and explains these can be used for which of the following? 1. Extras to the meal plan 2. To eat an extra slice of toast without counting its calories 3. To add servings in any major food group 4. The fat and sugar in your plan

1. Extras to the meal plan

The nurse educator is assessing learning about coenzymes with nursing students. A correct understanding of function of these molecules has been demonstrated when which of the following examples of the function of coenzymes is given? 1. Folacin helps with protein synthesis. 2. Vitamin C prevents rancidity in foods. 3. Antioxidants prevent cell oxidation. 4. Vitamin D helps absorption of calcium.

1. Folacin helps with protein synthesis.

The nurse educator is introducing functional to a group of nursing students. Which of the following is best way for the nurse to describe functional foods? 1. Functional foods are those that have components that appear to enhance health. 2. Functional foods are those that treat specific health concerns. 3. Functional foods are those that have been altered to produce medicinal benefits. 4. Functional foods are those that are nutritionally complete.

1. Functional foods are those that have components that appear to enhance health.

A student in the nutrition class is reporting on the availability of nutrition information. According to the student's report, where do most Americans get their information on nutrition? 1. Health-care professionals 2. Nutrition bloggers 3. Friends and family 4. TV food experts

1. Health-care professionals

The nurse is providing client teaching regarding the symptoms of gastroesophageal reflex disease (GERD). Which of the following teaching points should be included? Select all that apply. 1. Heartburn 2. Indigestion 3. Regurgitation 4. Pain that decreases when you bend over 5. Pain that increases when you stand up

1. Heartburn 2. Indigestion 3. Regurgitation

When providing information to nursing students about irradiation, which statement made by the nurse educator describes this process accurately? 1. Irradiation does not use heat. 2. Irradiation decreases the shelf life of a product. 3. Irradiation eliminates the need for safe food handling. 4. Irradiation hides spoilage.

1. Irradiation does not use heat.

The nurse is conducting nutrition counseling at the clinic. A client who is at risk for prostate cancer asks what he can do to reduce his risk. Which of the following is the most appropriate recommendation? 1. It is better to be as lean as possible without being underweight. 2. Take supplements that are known to prevent cancer. 3. Two alcoholic drinks per week are acceptable. 4. Transition to a vegetarian diet

1. It is better to be as lean as possible without being underweight.

The nurse is discussing organic labeling with the nutrition class when one of the participants asked about the criterion for labeling organic foods. What would be the best response? 1. It's on whether it meets criterion that defines the four official organic categories. 2. It's based on what natural pesticides are used in the growing of the foods. 3. It's based on whether the farm that produces the food has been certified organic. 4. It's based on the number of natural fertilizers used to produce the food.

1. It's on whether it meets criterion that defines the four official organic categories.

The nurse is providing education to a client who is learning to read labels to identify which products contain lactose. Which of the following are safe for the client with lactose malabsorption to consume? Select all that apply. 1. Lactalbumin 2. Lactate 3. Lactic acid 4. Casein

1. Lactalbumin 2. Lactate 3. Lactic acid 4. Casein

A nurse is seeing a client who reports she is concerned about free radicals. The client is seeking nutritional guidance to prevent the damage that can be caused by them. Which of the following foods can the nurse recommend for prevention? 1. Leafy green vegetables 2. Bananas 3. Starchy foods 4. Skim milk

1. Leafy green vegetables

A client in the prenatal clinic tells the nurse she is having trouble keeping her breakfast down and has waves of nausea throughout the day. Which of the following is the most appropriate nutritional intervention the nurse can suggest to the client? 1. Limiting fat intake 2. Drinking liquids with meals 3. Limiting carbohydrate intake 4. Increasing fluid intake

1. Limiting fat intake

The nurse is conducting client education with a client and his spouse before he is discharged from the hospital. The client is experiencing dysphagia. Which of the following is the best way for the nurse to describe honey-like liquids that can be included in the client's diet? 1. Liquids that can be eaten with a spoon but do not hold their shape 2. Liquids thickened to pudding consistency that need to be eaten with a spoon 3. All unthickened beverages and supplements 4. Liquids thicker than water but thin enough to sip through a straw

1. Liquids that can be eaten with a spoon but do not hold their shape

A client is concerned she is not getting enough vitamin D in her diet. The nurse provides some client education regarding sources of vitamin D. The nurse includes in the teaching that synthesis of vitamin D is NOT impaired by which of the following? 1. Low calcium intake 2. Sunscreens 3. Dense clouds 4. Aging

1. Low calcium intake

The nurse educator is discussing gastroesophageal reflux disease (GERD) with a group of nursing students. The students ask what recommendations should be given to a client with GERD to prevent symptoms. Which of the following is the nurse educator's best response? 1. Maintain a healthy body weight. 2. Drink fruit juices with meals. 3. Lie down 30 minutes after eating. 4. Eat 2 to 3 hours before bedtime.

1. Maintain a healthy body weight.

The nurse is conducting an assessment of a new client who has been using mega doses of nutritional supplements for several years. The nurse is aware that consistently high doses of many vitamins have no adverse effects; however, long-term exposure to synthetic folic acid may do which of the following? 1. Mask a deficiency of vitamin B12 2. Increase the need for other B vitamins 3. May increase the risk of heart disease 4. Can cause a temporary sensory neuropathy

1. Mask a deficiency of vitamin B12

The school nurse is discussing nutrition with a group of preteen students. After discussing the various options in the food groups, they discuss the way solid fats and added sugars can be hidden in food and the importance of reading labels. Which of the following are examples of food with hidden solid fats and/or sugar? Select all that apply. 1. Microwave popcorn 2. Banana chips 3. Salt 4. Plain beef jerky 5. Sweet rolls

1. Microwave popcorn 2. Banana chips 3. Salt 5. Sweet rolls

The nurse is caring for a hospitalized client with dumping syndrome. Which of the following foods should not be included in the diet menu for this client? 1. Milk 2. Mashed potatoes 3. Margarine 4. Lean meat

1. Milk

The nurse has given the nutrition class an assignment to list and designate the most common foodborne illnesses and their symptoms. What symptoms would the nurse expect to see designated by the students? Select all that apply. 1. Nausea 2. Sore throat 3. Muscle aches 4. Diarrhea 5. Fever

1. Nausea 4. Diarrhea 5. Fever

The triage nurse at the local emergency department is triaging a client who arrives complaining of vomiting, diarrhea, cramps, and mild fever. The client reports eating at a salad bar the day before. What should the nurse suspect as the causative agent? 1. Norovirus 2. Perfringens food poisoning 3. Listeriosis 4. Botulism

1. Norovirus

The nurse educator is discussing dysphagia and the National Dysphagia Diet in a nutrition class. The nurse educator asks the students to name foods the client could eat on level 2 of the diet. Which of the following is a correct response? 1. Pancakes with syrup 2. Enriched bread 3. Whole-grain cereals 4. Muffins

1. Pancakes with syrup

A mother is concerned for her 4-year-old daughter putting on too much weight. Which of the following are suggestions the nurse can make to help the mother? Select all that apply. 1. Read labels to reduce sugar intake. 2. Switch to sugar-free soda. 3. Keep healthy snacks on hand. 4. Increase her activity level. 5. Switch the child to a 1000-calorie diet.

1. Read labels to reduce sugar intake. 3. Keep healthy snacks on hand. 4. Increase her activity level.

The nurse asks a member of the nutrition class to state the basic messages of the Dietary Guidelines for Americans. What would they include in their answer? Select all that apply. 1. Reduce intake of saturated fats, sugars, and alcohol. 2. Manage weight by calorie balancing. 3. Do not worry about empty calories. 4. Always read the food label. 5. Prevent obesity with physical activity.

1. Reduce intake of saturated fats, sugars, and alcohol. 2. Manage weight by calorie balancing. 5. Prevent obesity with physical activity.

The nurse is discussing nutritional needs with a client who is considering pregnancy. The nurse correctly tells the client caloric needs typically start to increase at which point in the pregnancy. 1. Second trimester 2. First trimester 3. Third trimester 4. After giving birth

1. Second trimester

The nurse is preparing a client for discharge from hospital following a stroke. He is being advanced to level 3 dysphagia diet to assess his tolerance. Which of the following choices should the nurse recommend he choose for his dinner? Select all that apply. 1. Steamed green beans 2. Mashed potatoes 3. Meatloaf 4. Pineapple upside down cake 5. Pasta salad with raw vegetables

1. Steamed green beans 2. Mashed potatoes 3. Meatloaf

When treating a client for anorexia nervosa (AN), it is important to know what diet interventions work best. Which diet interventions are appropriate for clients with anorexia nervosa? Select all that apply. 1. Supplement with vitamins and minerals 2. Providing small, frequent feedings 3. Increasing sodium intake 4. Avoiding caffeine intake 5. Emphasizing high fiber foods

1. Supplement with vitamins and minerals 2. Providing small, frequent feedings 4. Avoiding caffeine intake

For a client with improving dysphagia the speech therapist has recommended moving the client to nectar-like liquids. Which of the following liquids can the nurse permit on the client's diet tray? 1. Thin milkshakes 2. Ice cream 3. Watermelon 4. Clear juices

1. Thin milkshakes

One of the pregnant clients in a nutrition class voices concern about being constipated despite having very regular bowel movements in pre-pregnancy. Which of the following is the nurse's best response? 1. Try drinking hot water with lemon juice upon waking. 2. Eat vitamin-enriched white bread three times a day. 3. Eat iron-fortified cereal. 4. Drink prune juice with every meal.

1. Try drinking hot water with lemon juice upon waking.

The health-care provider has instructed the client to eat foods that will provide more vitamin E. Which of the following should the nurse recommend that the client eat? 1. Vegetable oils 2. Fortified milk 3. Egg yolks 4. Red meat

1. Vegetable oils

The nurse is seeing a client who reports she smokes cigarettes. According to the RDA, the nurse should discuss with the client increasing intake of which of the following vitamins? 1. Vitamin C 2. Vitamin E 3. Beta-carotene 4. Folate

1. Vitamin C

The nurse is aware that vitamins are organic compounds that differ in function and availability. When discussing properties of vitamins with a client, which of the following statements is accurate? 1. Vitamins are susceptible to destruction by factors such as heat, air, and light. 2. Vitamins exist in only one active form. 3. Vitamins provide energy. 4. Vitamins are composed of long chains of molecules linked together.

1. Vitamins are susceptible to destruction by factors such as heat, air, and light.

The nurse is providing education for a client being discharged from hospital on a cardiac diet. To assist the client in making food purchases that will have a long shelf life but retain nutrients, the nurse should suggest which of the following options? 1. Whole wheat flour 2. White fish 3. Low-fat dairy products 4. Sweet potatoes

1. Whole wheat flour

The nurse is caring for a client with dumping syndrome. While filling out the menu for the next day, the client asks what the best dessert would be. What is the best response? 1. Yogurt 2. Apple pie 3. Ice cream 4. Chocolate pudding

1. Yogurt

The nurse has been asked to speak on "Nutrition in the Foods We Eat" at a local town meeting. During the presentation, functional foods are explained. Which foods would be described as natural functional foods? Select all that apply. 1. Yogurt 2. Rice 3. Tomatoes 4. Corn 5. Garlic

1. Yogurt 3. Tomatoes 5. Garlic

As the nurse caring for a client with an 18% burns over the total body surface area (TBSA), how many grams of protein per kilogram of body weight should his diet provide? 1. 1.0 to 1.5 g 2.0 to 2.2 g 0.8 to 1.2 g 1.5 to 2.0 g

1.5 to 2.0 g

A client has requested ideas for menus on a low-fat diet. The nurse suggests which of the following menus? 3 oz of broiled chicken breast, 1/2 cup of baked potato with 1 tsp of margarine, 1/2 cup of steamed broccoli, and 1 cup of hot tea 1/2 cup of black beans with 1/3 cup of rice and tomatoes, 1 cup of salad with 1 tbsp of Italian dressing, 1/2 cup of steamed broccoli, 3/4 cup of mandarin oranges, and 1 cup of skim milk 3 oz of broiled steak, 1/2 cup of brown rice, 1 cup of green salad with 1 tbsp of low-fat ranch dressing, 1/2 cup of fresh strawberries, and 1 cup of steamed spinach 3 oz of tuna mixed with lettuce, tomato slices, chopped onions, grated carrots and 2 tbsp of Thousand Island dressing, 2 slices of whole wheat bread with 1 tbsp of butter, 2 oz of cheddar cheese sticks, and 1 cup of whole milk

1/2 cup of black beans with 1/3 cup of rice and tomatoes, 1 cup of salad with 1 tbsp of Italian dressing, 1/2 cup of steamed broccoli, 3/4 cup of mandarin oranges, and 1 cup of skim milk

For a client who is overweight and following a low-carbohydrate/high-fat diet, what would be an appropriate breakfast on this diet? 1. 2/3 cup of plain yogurt with 3/4 cup of blueberries, and 3 graham crackers 2. 1/2 cup of orange juice, 2 scrambled eggs cooked in 2 tsp of butter, and 1 oz of pork sausage 3. 1/2 cup of orange juice, 1 small banana, 1/4 cup of scrambled egg beaters, 2 slices of whole wheat toast, and 2 tsp of butter 4. 1/2 cup of orange juice, 1 oz of shredded wheat, 2 slices of whole wheat toast, 1 cup of skim milk, and 2 tsp of butter

1/2 cup of orange juice, 2 scrambled eggs cooked in 2 tsp of butter, and 1 oz of pork sausage

The nurse is discussing the benefits of exercise for a client with type 2 diabetes who uses insulin. The nurse should emphasize eating carbohydrate prior to commencing exercise when the blood glucose is which of the following? 200 mg/dL 130 mg/dL 100 mg/dL 110 mg/dL

100 mg/dL

The nurse is admitting a critical client to the surgical ICU. During the nutrition assessment, the nurse uses a simple method for estimating the client's total calorie needs based on his weight of 100 kg and the fact that he is obese. Approximately how many calories would this client require? 1500 to 1900 calories 1100 to 1400 calories 2500 to 2900 calories 2000 to 2400 calories

1100 to 1400 calories

During a visit to the maternal child clinic, a client asks the nurse when she can begin giving her baby whole milk. Which of the following is the nurse's best response? 2 years 9 months 12 months 6 months

12 months

The nurse is admitting a new client to the unit. The client is 17 years old and weighs 120 pounds. What is her basal metabolic rate (BMR) using the rule-of-thumb guideline? 1800 1200 600 900

1200

The client is a 32-year-old, well-developed, well-nourished female who was struck by a car while jogging. She has been admitted to the ICU. The nurse has determined she weighs 115 pounds. What will be her recommended calorie intake using the simple equation? 1400 to 1600 calories 1300 to 1500 calories 1200 to 1400 calories 1500 to 1700 calories

1300 to 1500 calories

The nurse is working with a 55-year-old female client who has chronic constipation. After evaluating the clients' food intake, it is determined that the client needs to ingest more fiber. What is the recommended daily intake of fiber for this client? 28 g/day per 1000 calories of intake 21 g/day per 1000 calories of intake 14 g/day per 1000 calories of intake 7 g/day per 1000 calories of intake

14 g/day per 1000 calories of intake

The client is beginning an exercise program for the health benefits and to aid in preventing weight gain. How many minutes per week should the client plan on performing moderate-intensity physical activity to meet the recommended number of minutes? 150 minutes/week 90 minutes/week 120 minutes/week 60 minutes/week

150 minutes/week

The nurse is counseling a young female adult who needs approximately 2000 cal/day to maintain her weight. The client has set a goal to lose 1 pound per week. The nurse should recommend that the client instead, consume how many calories per day to achieve her goal? 1600 cal/day 1000 cal/day 1800 cal/day 1500 cal/day

1500 cal/day

The nurse is admitting a critical client to the ICU. During the nutrition assessment, the nurse uses a simple method for estimating the client's total calorie needs based on his weight of 60 kg and the fact that he is not obese. Approximately how many calories would this client require? 1260 to 1360 calories 1800 to 2400 calories 1500 to 1800 calories 1320 to 1440 calories

1500 to 1800 calories

The nurse correctly tells the client the recommended calorie levels, based on gender, to produce a 1- to 2-pound weight loss a week is which of the following? 1800 cal/day for men and 1400 cal/day for women 2000 cal/day for men and 1500 cal/day for women 1600 cal/day for men and 1200 cal/day for women 1000 cal/day for men and 1300 cal/day for women

1600 cal/day for men and 1200 cal/day for women

The nurse is caring for a 25-year-old male who has sustained burns over 30% of his body. If the client weighs 68 kg, approximately how many calories per day should this client consume? 2100 calories 1900 calories 1700 calories 1500 calories

1700 calories

A mother is concerned her 15-year-old daughter is not very active. The mother wants to know how many calories a day should she feed her to keep her from gaining weight. 1800 calories 900 calories 1200 calories 1500 calories

1800 calories

The nurse is admitting a healthy, 25-year-old male to the unit for outpatient knee surgery. During the nutrition assessment, it is determined that he weighs 78.2 kg. Using the rule-of-thumb guideline for calculating BMR, what is the approximate BMR of this client? 2580 1892 1720 860

1892

The nurse is admitting a gentleman who weighs 75 kg and is 6 ft 6 in tall. What is this client's body mass index? 19 17 25 21

19

The nurse is helping the client order a low-fat meal. Which of the following is the best meal for this client? Potato soup, grilled cheese sandwich on whole wheat bread, 2% milk, and fresh peach slices 3 oz of grilled hamburger with low-fat cheese slices, tomato, onions, and 2 tbsp of mayonnaise and whole wheat bun; baked potato chips; and iced tea 3 oz of tofu burger; mashed potatoes with gravy; tossed salad with avocado slices, tomato, cucumbers, radishes, carrots, olives, onions, and 2 tbsp of ranch dressing; iced tea with lemon slices, and 1 cup chocolate ice cream 2 oz of broiled lean chicken, brown rice, steamed vegetables (broccoli, cauliflower, and carrots), tossed salad with raw vegetables and fat-free dressing, and fat-free milk

2 oz of broiled lean chicken, brown rice, steamed vegetables (broccoli, cauliflower, and carrots), tossed salad with raw vegetables and fat-free dressing, and fat-free milk

A young mother is concerned her 2-year-old daughter is gaining too much weight. Which of the following should the nurse recommend the mother to give daily when following a 1000 cal/day diet? Select all that apply. 3 cups dairy 2 oz protein 1 cup vegetables 4 oz grain 1/2 cup fruit 2 cups dairy

2 oz protein 1 cup vegetables 2 cups dairy

The nurse is helping a group of clients with diabetes learn how to use the food lists for diabetes. She asks them to name a food item that represents one carbohydrate choice. Which of the following correctly demonstrates one carbohydrate choice? Select all that apply. 2 tbsp of dried apricots 1 slice of whole wheat bread 1 cup of oatmeal 1/2 cup of peas 3/4 cup of strawberries 1/2 cup of potatoes

2 tbsp of dried apricots 1/2 cup of peas 1/2 cup of potatoes

In order to ensure there is a safe period of time prior to introducing new foods to infants, the nurse should inform the parents of a 6-month-old to wait how long between introducing new foods so that allergic reactions can be easily identified? 2 to 3 days 7 to 14 days 5 to 7 days 3 weeks

2 to 3 days

A nurse is reviewing the healthy heart recommendations from the Dietary Guidelines for Americans with a group of parents. The parents in the group most likely have children in which age range? 2 years to adult 1 year to adult 5 years to adult 10 years to adult

2 years to adult

A nurse is providing education to a client about the differences between fat-soluble and water-soluble vitamins. The client is correct in making which of the following statements after the teaching session? 1. "I need enough fat on my body to make sure water-soluble vitamins can be stored." 2. "I need to take water-soluble vitamins every day." 3. "I need to eat enough protein to help water-soluble vitamins to be carried through my blood." 4. "I need water-soluble vitamins more than I need fat-soluble vitamins."

2. "I need to take water-soluble vitamins every day."

When discussing healthy changes to the eating patterns with a client, the nurse should provide recommendations regarding eating less-than-healthful foods by making which of the following statements? 1. "Weekends can be your time to splurge each week." 2. "It's great to enjoy them on special occasions." 3. "You can eat them any time before 2 o'clock." 4. "They should be avoided entirely."

2. "It's great to enjoy them on special occasions."

The nurse is working with a group of clients who are learning how to read nutritional labels on their favorite foods. The clients are learning the definitions of various terms that are used. The clients show they understand when one makes which of the following comments? 1. "Lean means the meat has less than 6 g of saturated fat." 2. "Reduced means the product has at least a 25% reduction in a nutrient when compared to a regular product." 3. "Good source means this product will provide 25% of the daily value for a nutrient." 4. "Very low means there is not much fat in this product."

2. "Reduced means the product has at least a 25% reduction in a nutrient when compared to a regular product."

The nurse is working with a 23-year-old client to develop a menu plan for a 2000-calorie food plan. After presenting MyPlate options, the client decides on a menu for the day. Which of the following is the best option for this client? 1. 2 slices white bread, 1 tbsp mayo, 2 slices bologna, 1 slice American cheese, 1 tsp mustard 2. 1 cup steamed spinach, 3 oz poached salmon with dill, 1 small apple, 1 cup hot tea sweetened with 1 tsp sugar 3. 2 cups spaghetti with meatball sauce, 2 cups salad with lettuce, carrots, celery, radishes, green peppers, green onions, and 3 tbsp ranch dressing 4. 1 cup yogurt with fruit, 1 cup spilt pea and ham soup, 1 cup coffee with creamer and sugar

2. 1 cup steamed spinach, 3 oz poached salmon with dill, 1 small apple, 1 cup hot tea sweetened with 1 tsp sugar

The client is a healthy 19-year-old college student who is interested in maintaining a healthy weight by following a 2000-calorie diet. How many servings of vegetables should he eat on a daily basis? 1. 21/2 cups 2. 3 cups 3. 31/2 cups 4. 2 cups

2. 1/2 cups

What are the nutritional recommendations for calcium during a normal pregnancy? 1. 1400 mg/day 2. 1000 mg/day 3. 800 mg/day 4. 1200 mg/day

2. 1000 mg/day

The nurse has assessed a 28-year-old female client as being at risk for having an unrealistic weight loss goal. The nurse should correctly inform the client that a realistic caloric intake for her is which of the following? 1. 1500 to 1800 cal/day 2. 1200 to 1500 cal/day 3. 1800 to 2100 cal/day 4. 1000 to 1200 cal/day

2. 1200 to 1500 cal/day

While discussing the "Nutrition Facts" label with a community diet and nutrition class, the nurse discusses the Percent Daily Value. The nurse correctly states the Percent Daily Value (%DV) listed on the "Nutrition Facts" label based which of the following? 1. Serving size of 1 cup 2. 2000-calorie diet 3. Percentage of carbohydrate, protein, and fat 4. 150-pound individual

2. 2000-calorie diet

Although many people may not be meeting recommended nutritional guidelines for optimal health, it is not always necessary for people to take a vitamin supplement. Which of the following individuals is least likely to need a vitamin supplement? 1. An elderly woman living alone 2. A moderately active adult 3. A pregnant teenager 4. An adult female on a low-calorie diet

2. A moderately active adult

The client is adhering to the Acceptable Macronutrient Distribution Ranges (AMDRs) in his diet. He knows that adhering to the AMDRs has been associated with which of the following? 1. A reduced rate of breast cancer 2. A possible decrease in chronic disease risk 3. A reduced risk of type 1 diabetes mellitus 4. Weight loss

2. A possible decrease in chronic disease risk

A supplement taken by many Americans to aid in lowering serum cholesterol levels is garlic. The nurse should provide additional education about garlic to clients who are prescribed which of the following? 1. Vitamin E supplements 2. Anticoagulants 3. Antidepressants 4. Digitalis

2. Anticoagulants

The client is concerned that her milk may not be providing adequate nutrition for her infant due to inadequate diet. The nurse suspects that the client may not be able to provide enough of which of the following nutrients through breast milk? 1. Fat content 2. Calorie content 3. Calcium content 4. Protein content

2. Calorie content

A client who has been identified as having excessive gestational weight gain (GWG) in her pregnancy has come to the prenatal clinic for a visit with a nurse. When providing education to the client about risks associated with GWG, which of the following should the nurse include? 1. Constipation 2. Cesarean section 3. Pica 4. Hypotension

2. Cesarean section

The nurse is asked to explain the concept of variety within a major food group. Which would be the best example to give to illustrate the concept of variety within a major food group? 1. Margarine, butter, mayonnaise 2. Chicken, black beans, walnuts 3. Hamburger roll, waffle, saltine crackers 4. Orange juice, fresh grapefruit, lemonade

2. Chicken, black beans, walnuts

The nurse is teaching the high school health class the benefit of including milk in the diet to prevent which of the following health complications 1. Low blood calcium levels 2. Compromise of bone integrity 3. Tetany reaction 4. Impaired protein synthesis

2. Compromise of bone integrity

During the nutrition class, the nurse asks one of the students to describe the Estimated Average Requirement (EAR). The student has answered correctly by stating which of the following? 1. EAR is the amount of a nutrient that is recommended to prevent chronic illness. 2. EAR is the amount of a nutrient that is estimated to meet the requirement of half of healthy people in a lifestyle or gender group. 3. EAR is the amount of a nutrient that meets the needs of 98% of all people. 4. EAR is the amount of a nutrient that meets the needs of all healthy people in a specific population.

2. EAR is the amount of a nutrient that is estimated to meet the requirement of half of healthy people in a lifestyle or gender group.

What additional suggestions should the nurse make? 1. Increase fiber intake. 2. Eat dry crackers between meals. 3. Drink green tea with each meal. 4. Keep hard candy available.

2. Eat dry crackers between meals.

The nurse is counseling a clinic client with dumping syndrome. Which of the following would be the most effective intervention for the nurse to suggest to help the client manage symptoms? 1. Limit fat intake. 2. Eat small, frequent meals. 3. Choose high-fiber cereals. 4. Drink fluids with meals.

2. Eat small, frequent meals.

The nurse is preparing the client to make the necessary dietary changes from pregnancy to lactation. Which of the following is most important for the nurse to include in client teaching? 1. Thirst is not a reliable indicator of need. 2. Even if a mother has adequate fat stores, calorie intake should be increased 3. If she does not consume enough calcium, her milk will be calcium deficient. 4. The intake does not need to be restricted because the caffeine does not enter breast milk.

2. Even if a mother has adequate fat stores, calorie intake should be increased

A client voices concerns about preventing neural tube defects during her pregnancy. The nurse knows the client should be taking which of the following supplements to prevent these problems? 1. Vitamin B6 2. Folic acid 3. Niacin 4. Riboflavin

2. Folic acid

A nurse is assessing a client who has a peptic ulcer. The client asks what is the most likely cause of his ulcer. Which of the following is the nurse's best response? 1. Nonsteroidal anti-inflammatories 2. Helicobacter pylori infection 3. Alcohol 4. Spicy foods

2. Helicobacter pylori infection

One of the major messages conveyed by the Dietary Guidelines for Americans includes the consumption of alcohol. What is the message about alcohol? 1. Drink alcohol to ward off heart disease. 2. If you drink alcohol, drink in moderation. 3. Drink alcohol to lessen your risk of colon cancer. 4. If you drink alcohol, include it in your empty calories.

2. If you drink alcohol, drink in moderation.

The nurse has a client in the clinic who reported nausea and vomiting over several months. For which of the following health complications should the client be assessed further? 1. Rhinovirus 2. Intestinal obstruction 3. Increase in gastric acid secretion 4. Increase in digestive enzyme activity

2. Intestinal obstruction

When describing the overall goals of Dietary Guidelines for Americans to a client who wants to make changes in his eating pattern, which of the following is the best way for the nurse to describe this initiative? 1. It is an initiative aimed at improving the health of people with chronic disease. 2. It is an initiative aimed at helping people make higher quality food choices. 3. It is an initiative aimed at helping people chose fat-free options most. 4. It is an initiative aimed at maximizing health disparities among Americans

2. It is an initiative aimed at helping people make higher quality food choices.

The nurse is discussing the thermic effect of food on the body. When explaining how energy is used by the body for this purpose, which of the following information should the nurse include? 1. It can be precisely measured. 2. It is influenced by the portions sizes of each meal. 3. It must be accounted for when calculating calories spent in day. 4. It is estimated to be about 1/4 of total calorie intake.

2. It is influenced by the portions sizes of each meal.

While learning about the "Nutrition Facts" label, the client sees the term "very low" on a food label. What does this mean? 1. It refers to meat and poultry products with less than 10 g of fat. 2. It refers to sodium only. 3. The product has one-third fewer calories than a comparable product. 4. The product contains virtually none of the specified nutrient.

2. It refers to sodium only.

When providing teaching about H. pylori to a client with a peptic ulcer, how should the nurse best explain why this problem occurs in susceptible people? 1. The stomach produces less acid leading to food eroding the stomach. 2. It releases an enzyme that decreases stomach mucus. 3. A layer of the stomach is eroded. 4. It increases stomach acid so much that the stomach wall is destroyed.

2. It releases an enzyme that decreases stomach mucus.

The nurse is developing a pamphlet for the local public health department. The topic of the pamphlet is preventing foodborne illnesses. What information should be included in the pamphlet about keeping foods safe? Select all that apply. 1. Foods are safest frozen. 2. Keep foods separated. 3. Only consume irradiated foods. 4. Cook foods to the correct temperature. 5. Wash your hands often.

2. Keep foods separated. 4. Cook foods to the correct temperature. 5. Wash your hands often.

A nurse is teaching a class on nutrition and is discussing what clients need to know about taking dietary supplements. Which of the following information should the nurse include? Select all that apply. 1. Take supplements and prescribed medications together. 2. Look for FDA resources on the internet. 3. Look for evidence to determine safety. 4. Take one supplement at a time. 5. Consider the use of natural supplements for infants.

2. Look for FDA resources on the internet. 3. Look for evidence to determine safety. 4. Take one supplement at a time.

The nurse educator is discussing gastroesophageal reflux disease (GERD) with a group of nursing students. The students ask what recommendations should be given to a client with GERD to prevent symptoms. Which of the following is the nurse educator's best response? 1. Lie down 30 minutes after eating. 2. Maintain a healthy body weight. 3. Drink fruit juices with meals. 4. Eat 2 to 3 hours before bedtime.

2. Maintain a healthy body weight.

The nurse is sharing knowledge about food biotechnology with a nutrition class. During a learning activity, the nurse asks participants to identify a major concern voiced by critics of food biotechnology. Which of the following would be the best answer? 1. Limitations to production of new food varieties as result of genetic engineering 2. Major allergens can be introduced into foods that do not usually cause an allergic reaction. 3. Shelf lives of foods are decreased leading to lower freshness. 4. Genetically modified crops produce significantly less yield than organic crops.

2. Major allergens can be introduced into foods that do not usually cause an allergic reaction.

The nurse is teaching a nutrition class for nursing students. They are given an assignment of planning a 1-day menu for someone with gastroesophageal reflux disease (GERD). What would be an acceptable main meal for a client who has GERD? 1. Curried rice 2. New York strip steak 3. Enchiladas with red sauce 4. Chili casserole

2. New York strip steak

When conducting an assessment of a new client, the nurse asks if the client uses any supplements. What is the approximate percentage of American adults who take vitamin supplements based on their beliefs about their health? 1. One-third to one-half 2. One-half to two-thirds 3. More than two-thirds 4. Less than one-third

2. One-half to two-thirds

Bulimia nervosa (BN) is an eating disorder where nutritional counseling focuses on identifying and correcting food misinformation and fears. Which of the following is true about bulimia? Select all that apply. 1. Bulimia is harder to treat than anorexia. 2. People with bulimia lack of sense of control over eating. 3. Bulimia has a higher mortality rate than anorexia nervosa. 4. People with bulimia experience weight fluctuations and tend to be of normal or slightly above normal weight. 5. People with bulimia recognize the behavior is abnormal.

2. People with bulimia lack of sense of control over eating. 4. People with bulimia experience weight fluctuations and tend to be of normal or slightly above normal weight. 5. People with bulimia recognize the behavior is abnormal.

When teaching a class about the role of nutrition in preventing illness transmitted through viruses and bacteria, a nurse should include a discussion regarding foods that contain which of the following? 1. B vitamins 2. Phytonutrients 3. Water-soluble vitamins 4. Fat-soluble vitamins

2. Phytonutrients

The nurse is caring for a client who is recovering from a stroke. The client's dysphagia has improved and the speech therapist has recommended changing this client's diet from level 1 of the National Dysphagia Diet to level 2. What foods would now be included in this client's diet? Select all that apply. 1. Coconut 2. Poached eggs 3. Soft canned or cooked fruit 4. Peanut parfait 5. Well-moistened pancakes with syrup

2. Poached eggs 3. Soft canned or cooked fruit 5. Well-moistened pancakes with syrup

When assessing a client who is planning to become pregnant, the nurse knows which of the following factors puts the client at risk for poor nutritional status during pregnancy? Select all that apply. 1. Lacto-ovo vegetarian 2. Pre-pregnancy BMI of 17 3. Heavy menstruation in pre-pregnancy 4. Exercise of 30 minutes per day 5. Living in a remote area

2. Pre-pregnancy BMI of 17 3. Heavy menstruation in pre-pregnancy 5. Living in a remote area Correct answer

The nurse is presenting a workshop on preventing foodborne illness. Which of the following is one point that should be included in the workshop? 1. Promptly freezing meat after purchasing 2. Preventing cross-contamination of raw and cooked foods 3. Ensuring vaccinations for food service personnel are up to date 4. Avoiding raw food consumption

2. Preventing cross-contamination of raw and cooked foods

Because functional foods appear to enhance health or prevent disease, which natural functional food may help to prevent stroke? 1. Yogurt 2. Purple grape juice 3. Garlic 4. Green tea

2. Purple grape juice

The nurse explains that a label should include some specific facts now required by law for the labeling of dietary supplements. Which of the following are some required facts? Select all that apply. 1. Efficacy of the product 2. Quantity of the product 3. Quality of the product 4. Purity of the product 5. Intended use of the product

2. Quantity of the product 3. Quality of the product 4. Purity of the product

The client is learning to read the "Nutrition Facts" label when shopping for food. Information that appears on the "Nutrition Facts" label is specific for which of the following? 1. A 1800-calorie diet 2. Size listed 3. Size eaten 4. A 2500-calorie diet

2. Size listed

The nutrition class is learning about food supplements. Which is true about how food supplements are regulated by the U.S. Food and Drug Administration (FDA)? 1. The FDA regulates dietary supplements as over-the-counter medications. 2. The FDA regulates dietary supplements as foods. 3. The FDA regulates dietary supplements as herbs. 4. The FDA regulates dietary supplements as drugs.

2. The FDA regulates dietary supplements as foods.

The nurse has given the nutrition class the assignment of creating a list of interventions that will benefit a person with gastroesophageal reflux disease (GERD). What intervention should be expected on the list? 1. The client should use peppermint after every meal to settle the stomach. 2. The client should lose weight if overweight. 3. The client should avoid items that increase lower esophageal sphincter pressure. 4. The client should include bedtime snacks in the diet.

2. The client should lose weight if overweight.

The nurse is caring for a 21-year-old female with intractable nausea and vomiting. The nurse is discussing possible causes for this problem. Which of the following possible causes should the nurse include? 1. Growths on the inner wall of the bowels 2. The gallbladder is not completely emptying 3. Consumption of spicy foods 4. An increase in digestive enzyme production

2. The gallbladder is not completely emptying

The nurse is seeing a client preoperatively for surgery to repair a duodenal ulcer. The client asks for ways to prevent more ulcers, which of the following is the nurse's best response? There is evidence that a diet without caffeine may reduce the risk of duodenal ulcers. 1. Limit the intake of alcohol. 2. There is evidence that a diet high in soluble fiber may reduce the risk of duodenal ulcers. 3. Leave chocolate out of the diet.

2. There is evidence that a diet high in soluble fiber may reduce the risk of duodenal ulcers.

The regulation of dietary supplements falls to the FDA. The nurse is providing information about this regulation in a nutrition class. The nurse correctly explains that the difference between the regulation of dietary supplements and drugs is which of the following? 1. There is scientific proof of the efficacy of the product. 2. They are regulated by the FDA as foods. 3. There is scientific proof of an optimum dosage. 4. Dosage is standardized among manufacturers.

2. They are regulated by the FDA as foods.

The clinic nurse is conducting an education session for clients who are taking anticoagulants. Which of the following important information should the nurse include when providing teaching to the clients? 1. They should take the vitamin D at the same time each day. 2. They should take the vitamin K at the same time each day. 3. They should avoid all sources of vitamin D. 4. They should avoid all sources of vitamin K.

2. They should take vitamin K at the same time each day.

During a nutrition class, students learn that vitamins can exist in more than one form and have more than one function. Of which of the following vitamins is this true? 1. Thiamin 2. Vitamin A 3. Riboflavin 4. Vitamin B12

2. Vitamin A

The nurse is assessing a client's knowledge regarding a recommended eating plan. When the nurse asks the client to correctly identify in which situation an eating plan provide optimal amounts of all nutrients, the client should reply which of the following? 1. When all foods offer the possibility of improving body function 2. When most of the food choices are considered identified as " source" 3. When the food has been cooked long enough to kill foodborne illness 4. When the portion size is large enough to include a variety of nutrients

2. When most of the food choices are considered identified as " source"

The nurse is admitting a healthy 35-year-old woman to the outpatient surgery department for a day surgical procedure. She consumes approximately 2000 cal/day. What is her recommended intake of water? 2000 mL of fluid per day 2500 mL of fluid per day 1000 mL of fluid per day 1500 mL of fluid per day

2000 mL of fluid per day

The client is a 25-year-old female student being seen for a physical examination. If this client is 5 ft 4 in tall and weighs 125 pounds, what is her BMI? 21 17 19 23

21

The nurse is teaching a nutrition class on how to estimate the total number of calories expended by a person per day according to the rule-of-thumb formula. What is the estimated total calorie expenditure for a 160-pound man who does little activity? 1920 3220 2112 2288

2112

A woman is being admitted to the outpatient surgery department. The nurse discovers she weighs 130 pounds and is 5 ft 5 in tall. What is her body mass index? 22 19 15 17

22

The nurse is conducting an assessment on a new client who is being admitted to the long-term care facility. The client is a 69-year-old gentleman who is moderately active. What is the recommended calorie per day intake? 2200 cal/day 2400 cal/day 2000 cal/day 2600 cal/day

2200 cal/day

A client has been told to monitor his sodium intake. What is the Upper Limit (UL) set for adults for sodium Adequate Intake (AI)? 3000 mg 2300 mg 1700 mg 2500 mg

2300 mg

The client is being admitted for outpatient surgery. During the assessment, the nurse notes the client is 6 ft 3 in tall and weighs 97 kg. What is this client's BMI? 27 24 33 30

27

The client is a 32-year-old female whose pre-pregnancy body mass index (BMI) is 18. She is pregnant with a single fetus. Which amount of weight gain should the nurse recommend as healthy? 1. 25 to 35 pounds 2. 28 to 40 pounds 3. 11 to 20 pounds 4. 15 to 25 pounds

28 to 40 pounds

The client has an active 15-year-old boy. To maintain his weight, how many calories may he need per day? 1800 calories 3200 calories 2000 calories 2800 calorie

2800 calorie

The client is learning to make healthy choices in the food she serves her family. Based on a 2000-calorie diet, how many servings of dairy products should she plan for her two adolescent children, her husband, and herself on a daily basis? 1. 2 cups 2. 31/2 cups 3. 3 cups 4. 21/2 cups

3 cups

When providing nutrition therapy for a client who has a diagnosis of bulimia nervosa, which lunch would be best for an 18-year-old client who is being treated for bulimia? 1. 6 chicken nuggets, 1/4 cup of ketchup, 4 oz cup of applesauce, and one 4 in. slice of pumpkin pie with whipped cream 2. 3 oz of grilled swordfish, 1/2 cup of steamed broccoli, 1/2 cup of rice pilaf, and 4 oz of hot tea 3. 6 oz of chicken noodle soup, 6 saltine crackers with 2 tbsp of butter, 3/4 cup of ice cream, and 12 oz of diet soda 4. 3 oz of hamburger, lettuce leaf, tomato slice, 1 slice of onion, 1 oz of processed cheese, 1 hamburger bun, 6 oz of French fries, and one 8-oz strawberry milkshake

3 oz of grilled swordfish, 1/2 cup of steamed broccoli, 1/2 cup of rice pilaf, and 4 oz of hot tea

The nurse should identify which lunch is best for a healthy 65-year-old who lives independently? 3 oz of tuna on whole wheat bread, 1 medium apple, and 8 oz of 2% milk 3 oz of hamburger, lettuce leaf, tomato slice, 1 slice of onion, 1 oz of processed cheese, 1 hamburger bun, 6 oz of French fries, and one 8-oz strawberry milkshake 6 oz of chicken noodle soup, 6 saltine crackers with 2 tablespoons of butter, 3/4 cup of ice cream, and 12 oz of diet soda 6 chicken nuggets, 1/4 cup of ketchup, 4 oz cup of applesauce, and one 4-inch slice pumpkin pie with whipped cream

3 oz of tuna on whole wheat bread, 1 medium apple, and 8 oz of 2% milk

Which lunch is best for a 10-year-old who is active? 6 oz of chicken noodle soup, 6 saltine crackers with 2 tbsp of butter, 3/4 cup of ice cream, and 12 oz of diet soda 6 chicken nuggets, 1/4 cup of ketchup, 4-oz cup of applesauce, and one 4-in. slice of pumpkin pie with whipped cream 3 oz of tuna on whole wheat bread, 1 medium apple, and 8 oz of 2% milk 3 oz of hamburger, lettuce leaf, tomato slice, 1 slice of onion, 1 oz of processed cheese, 1 hamburger bun, 6 oz of French fries, and one 8-oz strawberry milkshake

3 oz of tuna on whole wheat bread, 1 medium apple, and 8 oz of 2% milk

As children grow, they generally consume more calories, decide if they want to eat, and how much they want to eat. What is the rule-of-thumb guideline to determine an age-appropriate serving size for a 3-year-old? Equal to that of an adult 3 tsp 2 tbsp 3 tbsp

3 tbsp

The nurse is conducting nutrition counseling for a 21-year-old woman in the clinic area and discussing the credibility of information regarding organically grown foods. The nurse knows the client has an accurate understanding of organic foods when she makes which of the following statements? 1. "All fruits at the farmer's market are 100% organic USDA certified." 2. "There is a big nutritional difference between organic and conventional produce." 3. "There is no growth hormone in organically grown meats." 4. "I will save money if I buy organic produce and dairy products."

3. "There is no growth hormone in organically grown meats."

A nurse is seeing a client who is having difficulties understanding why it would be dangerous to take more vitamin D than indicated on the directions on the bottle. Which of the following responses by the nurse is best? 1. "Vitamin D is water soluble; excess is absorbed directly into the bloodstream." 2. "Vitamin D is fat soluble; excess can enhance blood thinning." 3. "Vitamin D is fat soluble; excess is stored." 4. "Vitamin D is water soluble; excess is excreted in the urine."

3. "Vitamin D is fat-soluble; excess is stored."

The client is now at 20 weeks of gestation. At conception, her weight was considered "normal." Approximately how much weight should she have gained by this time? 1. 6 pounds 2. 15 pounds 3. 12 pounds 4. 9 pounds

3. 12 pounds

The nurse is counseling a male client with a BMI of 31.8 about decreasing his caloric intake. What would be the best level of caloric intake for this client? 1. 1000 to 1200 cal/day 2. 1300 to 1700 cal/day 3. 1200 to 1600 cal/day 4. 1100 to 1500 cal/day

3. 1200 to 1600 cal/day

A client has asked the nurse what is the recommended dosage for iron during pregnancy? Which of the following is the correct response? 1. 60 mg/day 2. 45 mg/day 3. 27 mg/day 4. 35 mg/day

3. 27 mg/day

The nurse is discussing weight gain with a group of pregnant women in a prenatal clinic. One of the women in the group has been measured with a body mass index (BMI) of 17.5. The nurse knows this client should gain how much weight during her pregnancy? 1. 40 to 50 pounds 2. 20 to 30 pounds 3. 28 to 40 pounds 4. 30 to 50 pounds

3. 28 to 40 pounds

The nurse is aware that particular populations are prone to vitamin deficiencies for various reasons. Why are older adults prone to vitamin D deficiency? 1. Their fixed incomes limit buying enough foods rich in vitamin D. 2. They excrete more vitamin D than younger adults do. 3. Aging reduces the kidneys' ability to activate vitamin D. 4. They are more likely to take prescription medicines that interfere with vitamin D absorption.

3. Aging reduces the kidneys' ability to activate vitamin D.

A nurse is seeing a client who has just found out she is 10 weeks pregnant. When discussing what to avoid, which of the following should the nurse emphasize most? 1. Caffeine 2. Aspartame 3. Alcohol 4. Acesulfame-K

3. Alcohol

B vitamins are known to be coenzymes and assist with various bodily reactions and functions. Not all coenzymes, however, are considered B vitamins. What substance is a coenzyme and is considered a non-B vitamin? 1. Isoniazid 2. Niacin 3. Carnitine 4. Beta-carotene

3. Carnitine

Based on the American Institute for Cancer Research, which of the following recommendations should the nurse include in an educational class on cancer prevention? 1. Eat less than 6 g of salt per day. 2. Limit saturated fat to 5% to 6% of total daily calories. 3. Develop a weight management strategy 4. Reduce the percentage of calories consumed from trans fats.

3. Develop a weight management strategy

The nurse is conducting client teaching for a new breastfeeding mother before she is discharged from hospital. Which of the following fluid intake recommendations should the nurse make to the client? 1. Drink at least 10 glasses of fluid every 24 hours. 2. Drink a glass of fluid every 2 hours around the clock. 3. Drink a glass of fluid every time the baby nurses and at every meal. 4. Drink a glass of fluid at every meal.

3. Drink a glass of fluid every time the baby nurses and at every meal.

American Cancer Society (ACS) and American Institute of Cancer Research (AICR) have both made recommendations for eating properly to prevent cancer. Which of the following are true for both guidelines? Select all that apply. 1. Do not use supplements to protect against cancer. 2. Choose whole grains instead of refined grains. 3. Eat a variety of food with emphasis on plant sources. 4. Limit the intake of red meats.

3. Eat a variety of food with emphasis on plant sources. 4. Limit the intake of red meats.

The nurse is assessing a client who is experiencing low appetite due to the side effect of medications prescribed. What is one of the recommendations the nurse can make to help decrease the client's anorexia? 1. Request the health-care provider to change the medications. 2. Take the medications with meals. 3. Eat meals that look good on your plate. 4. Drink a preferred beverage with each meal.

3. Eat meals that look good on your plate.

The nurse is assessing a client who has developed a complication of dumping syndrome. The nurse can recommend which of the following nonpharmacological treatments for the complication steatorrhea? 1. A high-dose vitamin C 2. Vitamin B12 injections 3. Fat-soluble vitamin supplements 4. A high-fiber diet

3. Fat-soluble vitamin supplements

The pregnant client tells the clinic nurse she is worried about neural tube defects in her baby. Which of the following nutritional sources should the nurse recommend to help clients prevent this fetal complication? Select all that apply. 1. Lean beef 2. Salmon 3. Folate supplement 4. Breakfast cereals 5. Lentils

3. Folate supplement 4. Breakfast cereals 5. Lentils

The nurse at an eating disorders clinic should work with a client who has been diagnosed with anorexia to focus of nutrition therapy goals on which of the following? 1. Increasing total grams of protein daily 2. Increasing total daily fat consumed 3. Increasing total number of daily calories 4. Increasing total weight gained daily

3. Increasing total number of daily calories

The nurse is working with a group of clients in a weight loss clinic. The nurse recognizes that the most successful diet plan is which of the following? 1. Low-carbohydrate diet 2. Balanced diet 3. Individualized diet 4. A low-fat diet

3. Individualized diet

The nurse is aware that the Dietary Guidelines for Americans are published every 5 years and are intended for whom? 1. Children aged 2 to 12 years 2. Infants and senior citizens 3. Individuals aged 2 years and older 4. Adults aged 18 years and older

3. Individuals aged 2 years and older

During a discussion on diet and nutrition at a community center, the nurse is asked to provide tips on preventing foodborne illness. Which of the following would be the best response? 1. Select unpasteurized dairy products. 2. Leave cooked food out until completely cooled before refrigeration. 3. Keep food preparation knives separate from each food. 4. Wash hands after cooking is complete.

3. Keep food preparation knives separate from each food.

1. The nurse is teaching a class on nutrition in the local nursing program. While discussing organically grown foods, it is explained that these foods are produced with which of the following? 1. Manufactured chemicals 2. Antibiotic additives 3. Little or no synthetic fertilizers 4. Artificial hormones

3. Little or no synthetic fertilizers

A nurse is assessing a client in the bariatric clinic. The client is seeking a healthier lifestyle by changing nutritional patterns. Which of the following is the most effective recommendation the nurse can make for this client? 1. Change several lifestyle behaviors at the same time. 2. Read the general healthy eating message. 3. Obtain nutrients from food, not supplements. 4. Follow the preplanned daily menus.

3. Obtain nutrients from food, not supplements.

The clinic nurse is conducting teaching regarding dietary supplements with an adolescent client who is an athlete at school. The client teaching should include which of the following? Select all that apply. 1. Be sure and take the supplement with any prescribed medication. 2. Obtain a prescription from your doctor for the supplement. 3. Read the dietary supplement label. 4. Discuss supplement use with your doctor. 5. Do not give supplements to children younger than 6 years of age.

3. Read the dietary supplement label. 4. Discuss supplement use with your doctor. 5. Do not give supplements to children younger than 6 years of age.

A client comes into the clinic complaining of arthritic symptoms. In reviewing the chart, the nurse notices that the client had been in the clinic approximately 3 weeks ago for treatment of foodborne illness. To which of the following pathogens was the illness most likely linked? 1. Botulism 2. Shigellosis 3. Salmonellosis 4. Listeriosis

3. Salmonellosis

Vitamins can be either fat soluble or water soluble. Their solubility determines many of their characteristics. What is one characteristic that is determined by a vitamin's solubility? 1. Strength 2. Availability 3. Storage 4. Function

3. Storage

The nurse provides a client with information about Dietary Reference Intakes (DRIs). The nurse correctly describes DRIs as which of the following? 1. The concepts of dietary-related chronic diseases 2. The concepts of balance and need 3. The concepts of probability and risk 4. The concepts of preventing nutritional deficiencies

3. The concepts of probability and risk

Thiamin acts as a coenzyme when the body metabolizes carbohydrates. It also has another important function. What is that other function? 1. Thiamin is important in the digestive process. 2. Thiamin is important in the metabolism of fats. 3. Thiamin is important in nervous system functioning. 4. Thiamin is important in liver functioning.

3. Thiamin is important in nervous system functioning.

The Dietary Guidelines for Americans is a program focused on healthy eating patterns as a whole rather than individual components such as food groups and nutrients. Which of the following is one of the goals of Dietary Guidelines for Americans? 1. To reduce the amount of calories consumed daily 2. To encourage decreased consumption of all dairy products 3. To use the guidelines as a framework to shift food choices 4. To follow one of the three styles of healthy eating patterns

3. To use the guidelines as a framework to shift food choices

When teaching the nutrition class about Dietary Reference Intakes (DRIs), the nurse would correctly state that this reference includes which of the following references sets? 1. Nutritional density values 2. The Eatwell Guide 3. Tolerable Upper Intake 4. Physical activity recommendations

3. Tolerable Upper Intake

The nurse is preparing a client for discharge who was admitted for pneumonia. The nurse is providing the client with strategies to increase his calorie intake at home. Which of the following suggestions should the nurse include? Select all that apply. 1. Use bronchodilator before eating. 2. Eat a soft diet. 3. Try high-calorie, high-protein nutrition formulas. 4. Avoid "gassy" foods. 5. Limit "empty" liquids with meals. 6. Rest before eating.

3. Try high-calorie, high-protein nutrition formulas. 4. Avoid "gassy" foods. 5. Limit "empty" liquids with meals.

Foodborne illnesses affect thousands of people every year. What is the major cause of foodborne illnesses? 1. Obtaining food from outdoor food vendors 2. Keeping foods at room temperature 3. Unsanitary food handling 4. Undercooking

3. Unsanitary food handling

The nurse is admitting a 42-year-old client with dysphagia following a head trauma. What fluid would be the best choice for this client on the nectar-like stage of the National Dysphagia Diet? 1. Bottled water 2. Iced coffee 3. Vegetable juices 4. Fruit juice

3. Vegetable juices

1. While explaining the irradiation of food to the nutrition class, the nurse gives the students list of foods allowed to be irradiated in the United States. Which of the following foods would be expected on this list? 1. Canola oil 2. Jelly 3. Wheat flour 4. Sweet potatoes

3. Wheat flour

A client is being admitted to the hospital unit with abdominal pain and nausea. During the assessment, the nurse discovers that the client has been taking supplements in manner in which the vitamins could result in acting more as drugs. When do vitamins act as drugs in the body? 1. When they are taken with certain foods 2. When they are taken in combination with other vitamins 3. When they are taken in megadoses 4. When they are prescribed by a physician

3. When they are taken in megadoses

MyPlate recommends Americans limit their intake of solid fats and added sugars to no more than 15% of total caloric intake. The nurse is seeing a client who reports he eats 2500 calories per day. What is the maximum recommended amount of caloric intake from sugar the client should consume in one day? 625 cal/day 500 cal/day 375 cal/day 250 cal/day

375 cal/day

The client is meeting with the nurse to discuss starting a weight loss program. The nurse explains that calories come from the foods we eat and the liquids we drink. The nurse should inform the client that there are how many calories per gram in carbohydrates? 4 cal/ g 12 cal/g 7 cal/g 9 cal/g

4 cal/ g

The mother of a 3-month-old infant asks the nurse in the breastfeeding clinic when the baby will be able to eat solids. Which of the following ages should the nurse respond? 2 to 3 months of age 6 to 9 months of age 9 to 12 months of age 4 to 6 months of age

4 to 6 months of age

Which of the following statements by a client who is obese in prepregnancy indicates to the nurse the client understands the nurse's teaching? 1. "I will likely gain about 50 pounds by the third trimester." 2. "I should have gained about 10 pounds in the first trimester." 3. "I should be gaining about 1.5 pounds per week." 4. "At the high end of the range, I can gain 20 pounds."

4. "At the high end of the range, I can gain 20 pounds."

The client is learning to make healthy choices in the food he eats by using MyPlate. Approximately what portion of the plate should be protein? 1. 35% 2. 15% 3. 45% 4. 25%

4. 25%

The nurse is helping a client develop some menus based on a 1200-calorie low fat diet. Which of the following is the better option based on the 1200 cal/day? 1. 2 oz of hamburger patty, 1 cup of cottage cheese, 1 cup of skim milk, 1 cup of mixed vegetables, and 2 slices of sourdough bread 2. 1 cup of green beans, 1 cup of raw carrot and celery sticks, 1 cup of raspberries with 1 teaspoon of honey, and 5 oz of fried chicken strips 3. 1.5 cups of corn, 1.5 cups of watermelon, 1 cup of milk, and 4 oz of buffalo chicken wings 4. 3 oz of baked chicken breast, 1 cup of strawberries, 1.5 cups of steamed broccoli, and 4 oz of rice pila

4. 3 oz of baked chicken breast, 1 cup of strawberries, 1.5 cups of steamed broccoli, and 4 oz of rice pila

The nurse is providing education to a group of clients about gastroesophageal reflux disease (GERD) that if goes untreated, can cause a variety of problems for the client. When reviewing the complications caused by untreated GERD, which of the following should the nurse include? 1. Gastric banding 2. Esophageal varices 3. Peptic ulcers 4. Adenocarcinoma

4. Adenocarcinoma

The clinic nurse is caring for a client who has dumping syndrome. After providing client teaching, what statement by the client tells the nurse the client understands the process? 1. An intermediate dumping reaction produces weakness, dizziness, and a rapid heartbeat. 2. An intermediate dumping reaction can occur 1 hour after eating. 3. An intermediate dumping reaction occurs because of a rapid absorption of carbohydrates. 4. An intermediate dumping reaction produces gas, abdominal pain, and diarrhea.

4. An intermediate dumping reaction produces gas, abdominal pain, and diarrhea.

The nurse is discussing the role of H. pylori in peptic ulcers. The nurse explains to a client which of the following is one of the treatments to treat this problem? 1. Antiemetics 2. Antihistamines 3. Antacids 4. Antibiotics

4. Antibiotics

Clients who need to be on antibiotics for a prolonged period of time may be at an increased risk of vitamin K deficiency for which of the following reasons? 1. Antibiotics inactivate vitamin K in the liver. 2. Antibiotics impair vitamin K absorption from foods. 3. Antibiotics cause loss of appetite, which lowers vitamin K intake. 4. Antibiotics kill intestinal bacteria that produce vitamin K.

4. Antibiotics kill intestinal bacteria that produce vitamin K.

The nurse is seeing a client who has a BMI of 36 along with type 2 diabetes and hypertension. The client has tried restrictive dieting unsuccessfully several times. The nurse knows the client should be considered for which of the following interventions? 1. Behavior modification 2. Phentermine (Fastin) 3. Orlistat (Xenical) 4. Bariatric surgery

4. Bariatric surgery

A client who is trying to reduce his blood pressure to prevent further cardiovascular risk is seeing the clinic nurse to plan his weight loss. Which of the following information should the nurse share with the client to keep the client motivated? 1. Cardiovascular risk is reduced when 10% of body weight is lost. 2. Blood pressure can improve with a 20% loss of body weight. 3. Cardiovascular risk is reduced when 15% of body weight is lost. 4. Blood pressure can improve with a 5% reduction of body weight.

4. Blood pressure can improve with a 5% reduction of body weight.

The nurse is assessing a client who is considering taking dietary supplements. What would be the best recommendation the nurse could give this client? 1. Be sure to read the manufacturer's information on the supplement as it is the most factual available. 2. Check any studies cited by the different manufacturers for results that tell you how to use the supplement. 3. Be sure to take supplements that complicate each other. 4. Check the FDA website for advisories on supplements to avoid.

4. Check the FDA website for advisories on supplements to avoid.

The nurse is discharging a client with dysphagia from hospital. Which of the following teaching points should the nurse include to help the client maximize oral intake as safely as possible? 1. Encourage liquids with meals. 2. Encourage the client to rest after meals. 3. Serve food and liquids hot. 4. Discourage consumption of alcohol.

4. Discourage consumption of alcohol.

The nurse is assessing a client who is experiencing low appetite due to the side effect of medications prescribed. What is one of the recommendations the nurse can make to help decrease the client's anorexia? 1. Take the medications with meals. 2. Drink a preferred beverage with each meal. 3. Request the health-care provider to change the medications. 4. Eat meals that look good on your plate.

4. Eat meals that look good on your plate.

A client is discussing the process of irradiation of food with the clinic nurse. The client is concerned that these food products are not safe to eat. The nurse can explain that irradiation has been determined to be safe and is sometimes referred to as which of the following? 1. Proton beam irradiation 2. Heat irradiation 3. Beta-ray sterilization 4. Electronic pasteurization

4. Electronic pasteurization

The nurse educator is teaching a class of nursing students about diet adherence in the obese client. The students are given an assignment to break into groups and make a list of strategies that can be used to encourage clients to adhere to their diets. In order to determine that teaching was effective, which strategy should be on every list? 1. Give the client a list of "potential diet buddies." 2. Give the client a list of ingredients for suggested recipes. 3. Give the client a list of restaurants with low-calorie menu items. 4. Give the client preprinted menus.

4. Give the client preprinted menus.

A nurse assessing a client notices he is taking large doses of niacin. The client informs the nurse that a health-care provider has instructed him to do so. Which of the following health issues is the most likely reason for this intervention? 1. High homocysteine levels 2. Osteomalacia 3. Bleeding gums 4. High LDL cholesterol

4. High LDL cholesterol

The nurse is discussing nutrition labels with a high school health class. Which of the following would be important to discuss the ingredients list? 1. Ingredients are listed by percentage of calories. 2. Ingredients are listed in descending order by amount. 3. Ingredients are listed alphabetically. 4. Ingredients are listed in descending order by weight.

4. Ingredients are listed in descending order by weight.

A client tells the clinic nurse that she is having cravings for nonfood items such as laundry starch, clay, and ice. The nurse anticipates this client is most likely at risk for which of the following? 1. Diarrhea 2. Excessive weight gain 3. Lead contamination 4. Iron deficiency

4. Iron deficiency

The nurse will need to discuss supplementation of vitamin B12 with a woman who has which of the following factors influencing her pregnancy? 1. Has a sedentary lifestyle 2. Is lactose intolerant 3. Has little sun exposure 4. Is a strict vegan

4. Is a strict vegan

The nurse is working with a group of clients in a weight loss clinic. One client asks if he is obese because it is part of his genetic makeup. Which of the following is the best way for the nurse to respond? 1. Obesity is caused mainly by low activity levels. 2. Obesity is largely determined by the food choices people make. 3. Having obese parents guarantees a person will be obese. 4. Lifestyle and environment along with genetics play a role in obesity.

4. Lifestyle and environment along with genetics play a role in obesity.

The nurse is providing nutrition information to a prenatal class. The nurse is correct in stating that compared with the Recommended Dietary Allowances of pregnancy, the RDA for lactation is which of the following? 1. Lower for vitamin D 2. Higher for calcium 3. Lower for protein 4. Lower for iron

4. Lower for iron

The nurse is teaching a nutrition class to a group of pregnant women in the community. A client in the class who is vegetarian asks what would be a good source of vitamin B12. Which of the following is the correct response? 1. Green leafy vegetables 2. Peanut butter 3. Vegetable oils 4. Milk

4. Milk

Riboflavin is said to be unique among the water-soluble vitamins. What makes riboflavin unique? 1. Meats and beans contribute the most riboflavin to the diet. 2. Milk and dairy products contribute the least riboflavin to the diet. 3. Meats and beans contribute the least riboflavin to the diet. 4. Milk and dairy products contribute the most riboflavin to the diet.

4. Milk and dairy products contribute the most riboflavin to the diet.

The nurse is conducting an educational session about overweight and obesity. When discussing the complications associated with obesity, which of the following should the nurse be certain to include? 1. Kidney stones 2. Seizure disorders 3. Intestinal polyps 4. Obstructive sleep apnea

4. Obstructive sleep apnea

The client with dysphagia is filling out his menu for the following day. What would be a good choice for breakfast for this client? 1. Pecan waffles 2. Peanut butter on toast 3. Bananas 4. Scrambled egg

4. Scrambled egg

The genetic engineering of our food supply has produced varied results. Which foods available in American supermarkets are most likely to have genetically engineered ingredients? 1. Milk and dairy products 2. Fortified wheat and oats 3. Tomatoes and corn oil 4. Soybean oil and corn syrup

4. Soybean oil and corn syrup

The nurse is seeing a client who has little or no appetite secondary to chronic anxiety. Which of the following goals should the nurse set with the client for increasing intake? 1. Stimulate the appetite to get the client to eat at least 50% of each meal. 2. Stimulate the appetite to get the client to eat at least 75% of each meal. 3. Stimulate the appetite to maintain three meals a day of oral intake. 4. Stimulate the appetite and ensure enough calories are consumed daily.

4. Stimulate the appetite and ensure enough calories are consumed daily.

The nurse works with several clients who routinely take nutritional supplements. The nurse is aware that which of the following statements about vitamins is true? 1. Because of the potential health benefits, beta-carotene supplements are routinely recommended. 2. Most Americans do not consume adequate amounts of vitamin C. 3. Most Americans do not consume adequate amounts of thiamin, riboflavin, and niacin. 4. Taking large doses of vitamin A is potentially harmful.

4. Taking large doses of vitamin A is potentially harmful.

The physician has recommended that a client take a vitamin supplement. What would the client look for when choosing a vitamin supplement? 1. Choose the highest price supplement 2. Choose a supplement that is "natural" 3. Look for the nutrient's Recommended Dietary Allowance (RDA) 4. The "Supplement Facts" label

4. The "Supplement Facts" label

It is recommended that people older than the age of 50 years take vitamin supplements, but it is important they take the proper type of supplement—namely, "50+" or "mature" multivitamin supplement formulas for which of the following reasons? 1. They contain less vitamin B12. 2. They contain more zinc. 3. They contain more calcium. 4. They contain less iron.

4. They contain less iron.

Inadequate gastric acid secretion secondary to gastric resection, use of medications that suppress gastric acid, or gastric infection with Helicobacter pylori is the most common cause of which of the 1. Riboflavin deficiency 2. Folic acid deficiency 3. Thiamine deficiency 4. Vitamin B12 deficiency

4. Vitamin B12 deficiency

The nurse is aware that vitamins have been found to have other functions than to just nourish the body. Vitamins are often used as food additives for specific purposes. Which vitamin is added to frozen fish to help prevent rancidity? 1. Vitamin E 2. Beta-carotene 3. Vitamin A 4. Vitamin C

4. Vitamin C

Older adults need adequate protein in their diet. What are the approximate protein needs per day for an older adult weighting 132 pounds (60 kg)? 36 g/day 48 g/day 24 g/day 44 g/day

48 g/day

The nurse is helping a client weight loss through calorie deficit. The nurse is correct in telling the client she needs a deficit of how many calories per day to produce a 1-pound weight loss per week? 1. 1000 calories 2. 500 calories 3. 250 calories 4. 2000 calories

500 calories

The nurse is providing nutritional education to a group of pregnant women in the second trimester. For one of the learning activities, the clients are asked to develop a lunch menu based on principles of healthy eating in pregnancy. Which of the following menus indicates the nutritional teaching has been effective? 2 oz beef; a medium-baked potato with 2 tbsp of sour cream, 2 tbsp of butter, and 2 oz of grated cheese; 1 cup of creamed corn; 6 oz of green tea; 3/4 cup of ice cream 6 pieces of sushi, 2 cups of saki, 4 oz of steamed vegetables 6 oz of swordfish, 1/2 cup tartar sauce, 2 slices of French bread with 11/2 tsp of butter on each, 3/4 cup of sweet peas, 1 4 in slice of apple pie with 1 scoop of vanilla ice cream 6 oz of chicken breast with tomato, 1 small apple, 1.5 oz low fat cheese, two 4 oz slices of whole wheat bread

6 oz of chicken breast with tomato, 1 small apple, 1.5 oz low fat cheese, two 4 oz slices of whole wheat bread

The nurse is teaching a nutrition class, and the subject for today is the basal metabolic rate (BMR). What percentage range would the nurse tell the students the BMR accounts for in the total energy requirements for most people? 25% 75% 60% 10%

60%

The nurse works on a burn unit and is admitting a 16-year-old female who weighs 50 kg. She has severe burns covering 65% of total body surface area. A high-protein diet is ordered. The dietitian figures the client's calorie and protein needs and orders approximately how many grams of protein per day? 75 to 100 g 50 to 75 g 100 to 150 g 150 to 200 g

75 to 100 g

The nurse is admitting a client who is diagnosed with type 2 diabetes mellitus. The physician writes an order for a body mass index (BMI) to be calculated. The nurse understands that a BMI is which of the following? 1. A calculation of an index of a person's weight in relation to height 2. A calculation of calorie intake necessary to maintain ideal weight 3. A calculation of an index of a person's ideal weight 4. A calculation of a person's prealbumin level

A calculation of an index of a person's weight in relation to height

While completing admission assessments, which of the following clients would the nurse identify to be considered at greatest nutritional risk? 1. A client who consumes one glass of wine at supper 2. A client who does not consume alcohol 3. A client who has four beers every evening 4. A client who likes occasional social alcohol

A client who has four beers every evening

The nurse needs to clarify information on the nutrition recommendations for managing diabetes when she hears one of the students state which of the following? A client with diabetes needs to keep total calories to weight maintenance levels. A client with diabetes needs to make sure their carbohydrate intake is consistent with the activities of the day. A client with diabetes needs to keep the amount of carbohydrate at each meal equal. A client with diabetes needs to adjust per-meal doses of insulin based on amount of carbohydrate to be consumed.

A client with diabetes needs to keep the amount of carbohydrate at each meal equal.

Which of the following changes in metabolic processes should indicate to the nurse that a client's hormone levels are declining? An increase in serum glucose levels Fluid retention An increase in urinary nitrogen excretion A decline in serum glucose levels

A decline in serum glucose levels

The nurse is working with a client who voices the desire to lose weight. When providing education about intake, the nurse should begin by explaining that a person who consistently consumes too many calories will be in which of the following? A positive energy balance A negative energy balance A negative caloric balance A positive caloric balance

A positive energy balance

When educating the client, the nurse understands that there can be some negativity associated with the term "diet." Many clients may view "diet" as another term for which of the following? 1. A positive experience 2. A health behavior 3. A short-term punishment to endure 4. A medical treatment

A short-term punishment to endure

One of the students in a nutrition class asks the nurse to explain the Acceptable Macronutrient Distribution Ranges (AMDRs). Which of the following is the best answer? 1. AMDRs provide specific ranges for essential nutrients. 2. AMDRs provide broad ranges for each energy nutrient with the aim of reducing risk of chronic disease. 3. AMDRs provide nonspecific ranges for major nutrients. 4. AMDRs provide the energy people need to consume to maintain balance.

AMDRs provide broad ranges for each energy nutrient with the aim of reducing risk of chronic disease.

The client is a 50-year-old male with cancer, who has been losing weight. The nurse suggests the client eat which of the following to increase his calorie intake? Substitute milk for water in recipes. Add butter to his hot foods. Spread peanut butter on apple slices. Use plain yogurt in place of sour cream.

Add butter to his hot foods.

The nurse educator has given the nutrition class the assignment of creating a list of interventions that will benefit a person with a colostomy. If teaching has been effective which of the following interventions should be on the student's list? Select all that apply. Add parsley to some dishes. Limit carbohydrates. Avoid chewing gum. Chew food thoroughly. Avoid foods that thicken stools.

Add parsley to some dishes. Avoid chewing gum. Chew food thoroughly.

The wife of a 69-year-old gentleman recovering from sepsis is frustrated her husband won't eat enough. She has been trying to feed him frequent small meals but he says he isn't hungry. Which of the following suggestions can the nurse make to increase his protein intake? Select all that apply. Add skim milk to milk. Add honey to fruit. Melt cheese on sandwiches, hot vegetables, and potatoes. Add chopped eggs to casseroles. Add marshmallows to hot chocolate. Use mayonnaise in place of salad dressing.

Add skim milk to milk. Melt cheese on sandwiches, hot vegetables, and potatoes. Add chopped eggs to casseroles.

When providing discharge teaching to a client recovering from a burn injury, which of the following should the nurse encourage to increase nutritional density of food? Scheduling treatments during mealtimes to encourage between meal snacking Consuming empty-calorie foods and beverages to promote intake Adding whole milk products to foods whenever possible Taking pain medications with meals to facilitate digestion

Adding whole milk products to foods whenever possible

The nurse is assessing the adolescent client with type 1 diabetes. The client has recently been experiencing fluctuations in her blood glucose levels due to increased physical activity. On which of the following interventions should the nurse help the client focus? Advanced carbohydrate counting Increasing caloric intake Using oral antidiabetic medications Decreasing use of insulin

Advanced carbohydrate counting

The nurse who is teaching a nutrition class should include that nutrition therapy for diabetes is appropriate for which of the following groups? Clients with type 1 diabetes Clients with gestational diabetes All clients with diabetes Clients with type 2 diabetes

All clients with diabetes

A client has been informed by a health-care provider that he may have a deficiency of copper. Provided this information, the nurse suspects which of the following? What mineral has the potential to cause a deficiency of copper in the body? Low intake of molybdenum Low intake of chromium An excessive intake of iron An excessive intake of zinc

An excessive intake of zinc

The nurse is teaching a class of health aides at a long-term care facility how to assess and document the nutritional intake of a client with decreased appetite. Which of the following teaching points should the nurse emphasize? Older adults can self-assess nutritional deficits. An individualized nutrition assessment is best. All older adults require a dietitian to assess nutrition. Only nurses are able to assess nutritional health.

An individualized nutrition assessment is best.

The nurse is conducting admission assessments on four new clients. Which client would be referred to the dietitian because of suspected malnutrition? 1. An adult client preoperative for hernia repair 2. An adult client postoperative for appendectomy 3. An older adult client who has muscle wasting 4. An older adult client who lives alone

An older adult client who has muscle wasting

The nurse is teaching a client with cholecystitis how to plan. What would be the best dessert for someone with cholecystitis? Angel food cake Apple pie Chocolate pudding Ice cream

Angel food cake

The nurse is aware malnutrition can occur because of the physiologic stress caused by respiratory distress. What factors may contribute to malnutrition that is cause by this problem? Select all that apply. Renal insufficiency Anorexia Dyspnea Abdominal bloating Diarrhea

Anorexia Dyspnea Abdominal bloating

The nurse is counseling a group of clients who have recently been diagnosed with celiac disease. One of the members in the group wants to know what is safe to use as a thickening agent in cooking. Which of the following would be a safe substitute? Durum flour Cracked germ Farina Arrowroot starch

Arrowroot starch

The nurse is teaching a high school health class on diet and exercise. Which of the following statements can she truthfully make? As the intensity of activity increases, the proportion of energy used increases. During the first 1 to 2 minutes of exercise, stored fat is the primary fuel. Physically fit people use more energy and store more fat. As the duration of activity increases, the proportion of fat used decreases.

As the intensity of activity increases, the proportion of energy used increases.

BMI is now being used to determine obesity; however, its use is not without controversy. Research has shown that ethnic differences exist in the relationship between BMI and health risks. Which ethnic group has been shown that the health risks of obesity can occur at a BMI lower than 30? Hispanic Americans Asian Americans Native Americans Black Americans

Asian Americans

The nurse is seeing a client with a colostomy, who reports some foods are creating odor problems. The nurse should inform the client that which of the following foods are odor-producing foods and should be avoided? Select all that apply. Asparagus Buttermilk Applesauce Peas Eggs

Asparagus Peas Eggs

The nurse has asked the mother of a preschool-aged child diagnosed with iron deficiency anemia to keep a food record to review daily intake for 1 week. During a follow-up visit, the nurse determines the preschooler's BMI and reviews a food record for which of the following reasons? Risks of obesity Risks for excessive TV watching Progression toward optimal BMI Assess the amount of milk consumed

Assess the amount of milk consumed

The clinic nurse is caring for a client with diabetes who has frequent episodes of hypoglycemia. What would be the best intervention for this client? Encourage the client to eat three large meals per day. Assess the client's motivation to adhere to treatment. Counsel the client to limit total carbohydrate intake to less than 100 g/day. Encourage the client to gain weight.

Assess the client's motivation to adhere to treatment.

The nurse is teaching a client about making better food choices to help her lose weight. Which of the following should the nurse encourage the client to choose? Select all that apply. Bologna and whole wheat bread sandwich Baked chicken without skin Apple slices Granola with milk Low fat fruit flavored yogurt Plain yogurt with fruit slices

Baked chicken without skin Apple slices Plain yogurt with fruit slices

The nurse is caring for a client with a new colostomy. When planning his discharge teaching, what food would be included that would help the client's control diarrhea? Buttermilk Eggs Banana Peas

Banana

A liberal diet approach is a holistic approach toward meeting client's nutritional needs. This approach takes into consideration which of the following? Select all that apply. Decreasing protein Based on individual needs Overall prognosis Limiting sodium Risk/benefit ratio

Based on individual needs Overall prognosis Risk/benefit ratio

A nurse is assessing a 6-month-old infant in a well-child visit. The nurse should expect to see development of which of the following feeding skills? Experiments with spoon but prefers to eat with hands Begins self-spoon feeding Swallows liquid but pushes most solid out Begins to eat mashed foods

Begins to eat mashed foods

The nurse obtains a height and weight at each well-child visit to determine the risk for obesity. All young clients who are not in a healthy weight range should be evaluated for which of the following? Select all that apply. Family dynamics Behavior risks Child neglect Medical risks Adequacy of food budget

Behavior risks Medical risks Adequacy of food budget

The nurse recognizes her client who has recently lost a lot of weight is at risk for refeeding syndrome. Which of the following will be important to monitor in this client? Intake and output Blood pressure Blood glucose levels Daily weight

Blood glucose levels

The nurse is working with a group of older adults to help them develop goals to achieve an optimal weight. The nurse provides information regarding current, evidence-informed practice for calculating ideal body weight. In doing so, the nurse will discuss which of the following? Body mass index Desirable weight chart Kilogram/meter calculations Basal metabolic rate

Body mass index

The nurse is conducting client education for a 21-year-old pregnant woman in the clinic. The client asks why she should choose to breastfeed rather than formula feed her baby. What would be the best response? The American Academy of Pediatrics recommends exclusive breastfeeding for the first 9 months of life. You will get down to your pre-pregnancy weight much faster if you breastfeed your baby. Breastfeeding is better for the kidneys when compared with formula feeding. Breast milk will consistently give the baby every vitamin essential for healthy development.

Breastfeeding is better for the kidneys when compared with formula feeding.

When asked by the health class asks the nurse which mineral is the most abundant in the body, what is the correct response? Iron Calcium Magnesium Phosphorus

Calcium

The nurse is helping a client plan menu choices. The client is currently recovering from dehydration from severe diarrhea. The client laboratory results showed hyponatremia and hypokalemia on admission. The client is now eating but refusing to eat bananas. The nurse suggests which of the following as a possible alternative? Pears Blueberries Strawberries Cantaloupe

Cantaloupe

Refeeding syndrome is a potentially fatal complication that can occur in critically ill clients. The nurse recognizes the seriousness of this complication as it can affect clients due to which of the following? Hypoglycemia Carbohydrate reintroduction Heart failure Hyperventilation

Carbohydrate reintroduction

The nurse is seeing an adolescent client diagnosed with type 1 diabetes. The client tells the nurse she worries about having a hypoglycemic episode while at school. Which of the following recommendations should the nurse make? Carry a package of hard candy in your pocket. Carry a high protein snack. Coordinate meals with insulin. Use an insulin pump.

Carry a package of hard candy in your pocket.

The nurse educator is discussing metabolic syndrome. The students correctly identify which of the following abnormalities are characteristic of metabolic syndrome? Select all that apply. Central obesity Low triglycerides Hypertension Low low-density lipoprotein (LDL) Low high-density lipoprotein (HDL)

Central obesity Hypertension Low high-density lipoprotein (HDL)

A client admitted to hospital with motor vehicle accident trauma is being discharged home to complete recovery. The diet that has been ordered is a high-calorie, high-protein diet in small frequent meals throughout the day. What would be the best sample menu for lunch for this client? A smoothie made with yogurt, instant breakfast, skim milk and strawberries, and a cookie Prime rib, baked potato with sour cream, a glass of white wine, and a piece of low fat cheesecake Chicken salad sandwich on a croissant, milk, ice cream with chocolate sauce, and whipped cream New England clam chowder, black coffee, and pears in 100% pear juice

Chicken salad sandwich on a croissant, milk, ice cream with chocolate sauce, and whipped cream

While discussing the various chemical properties of nutrients in the body, the class discusses the differences between anions and cations. What mineral is the major anion in the extracellular fluid? Chloride Choline Chromium Calcium

Chloride

When discussing the complications associated with obesity, the nurse should include which of the following? Select all that apply. Hypotension Cholecystitis Respiratory dysfunction Dyslipidemia Cystitis

Cholecystitis Respiratory dysfunction Dyslipidemia

The physician has recommended a low-fat diet for the client with cholecystitis. The client is frustrated because he likes to eat dairy products. Which of the following should the nurse suggest this client follow? Eliminate all milk products from his diet. Choose cheese with 5 g or less of fat. Choose dairy products that provide less than 1 g fat per serving. Switch to soy dairy products.

Choose dairy products that provide less than 1 g fat per serving.

The nurse is caring for a 68-year-old male client who is receiving enteral nutrition (EN). The nurse is aware there is an increased risk for refeeding syndrome in this client due to which of the following conditions noted in his past medical history? Bariatric surgery Morbid obesity Chronic alcoholism Cholecystectomy

Chronic alcoholism

While discussing nutrition with the group of clients, the nurse explains that some vitamins and minerals work together. What trace element is an essential component of vitamin B12? Selenium Cobalt Molybdenum Nickel

Cobalt

The nurse is assisting a client in making a menu for a low-fiber diet. Which would be the best choice for a breakfast menu? Hot tea, bagel, and 2 tablespoons of cream cheese Hot cocoa, shredded wheat, and 2% milk Coffee with cream and sugar, muffin, and 2 tablespoons of butter Apple juice, coffee, 2 poached eggs, 2 pieces of bacon, and white toast with butter

Coffee with cream and sugar, muffin, and 2 tablespoons of butter

The nurse is caring for a client with chronic, well-controlled diabetes who is post-op for a knee replacement. The nurse should inform the client that the diet offered in the acute care setting is which of the following? Carbohydrate counting diet High glycemic index diet Exchange lists for meals diet Consistent carbohydrate diet

Consistent carbohydrate diet

The nurse knows to provide which of the following information to family members of an older adult who are concerned about her hydration status? Water is the only effective fluid for hydration. Consume fluid at regular intervals throughout the day. Limit fluid intake to prevent incontinence. It is sufficient to drink when feeling thirsty.

Consume fluid at regular intervals throughout the day.

In order to ensure nutritional requirements for folic acid are met during pregnancy, clients need to do which of the following? Consume reinforced foods. Consume biotech foods. Consume enhanced foods. Consume fortified foods.

Consume fortified foods.

A nurse is recommending foods to avoid for a client who is pregnant. The nurse discusses caution when eating foods such as shark and swordfish for which of the following reasons? Consumption can lead to stunted fetal growth. Consumption can lead to spontaneous abortion. Consumption can lead to fetal malformations. Consumption can lead to neurologic abnormalities in the fetus.

Consumption can lead to neurologic abnormalities in the fetus.

The nurse is providing education to a client who asks about the long-term complications of diabetes. Which of the following interventions should the nurse recommend to significantly lower the risk of complications? Controlling blood glucose levels, blood pressure, and blood lipid level Controlling blood glucose levels, triglyceride level, and weight Controlling blood lipid level, carbohydrate intake, and blood glucose level Controlling protein intake, blood pressure, and blood lipid level

Controlling blood glucose levels, blood pressure, and blood lipid level

A family is meeting with the nurse to discuss their concern about their 78-year-old family member who is having difficulty chewing food. The client is refusing a pureed diet. Which of the following are examples of food the family can offer? Select all that apply. Fried potatoes Cooked carrots Ripe bananas Roasted chicken Soft melon

Cooked carrots Ripe bananas Soft melon

When conducting a nutritional screening with a client, the nurse tries to use terms other than "diet" when asking about the foods the client eats. Which term is not an appropriate alternative term to the word "diet"? 1. Eating pattern 2. Eating style 3. Counting calories. 4. The foods you eat

Counting calories.

Based on the review of a client's food log, the nurse suspects a client has lactose malabsorption. This is likely because the client experiences which of the following symptoms shortly after drinking milk? Bloody stools Cramping Vomiting Nausea

Cramping

The nurse is providing education to a group of clients who are recently diagnosed with celiac disease. The nurse should include which one of the following foods that needs to be avoided by clients with this disease? Cream of rice Corn flakes Cream of wheat Puffed rice

Cream of wheat

To be awarded the USDA organic seal, a food must adhere to strict national standards. What criteria must organic labeling meet? 1. Criteria that define the three official organic categories 2. Criteria that define the two official organic categories 3. Criteria that define the one official organic category 4. Criteria that define the four official organic categories

Criteria that define the four official organic categories

The nurse is providing education to a group of clients recently diagnosed with diabetes. To test client knowledge, the nurse asks which of the following can cause hypoglycemia. Which of the following would be an acceptable answer? Delaying mealtime Too much food Stress Acute illness

Delaying mealtime

A nurse has attended a seminar covering the objectives of the Dietary Guidelines for Americans. Upon returning to the clinic, the nurse is asked what the clinic could do differently to meet the objectives. Which of the following would be an appropriate answer? 1. Recommend a vegetarian diet to clients who eat meat. 2. Discuss with clients what to eat rather than what not to eat. 3. Tell clients to weigh themselves regularly. 4. Encourage all clients to achieve a caloric intake of 1600 cal/day.

Discuss with clients what to eat rather than what not to eat.

The nurse educator discusses distribution of body fat is the most likely indicator of which of the following? Body composition Obesity Disease risk Chronic disease process

Disease risk

The nurse knows which of the following contributing factors puts adolescents at risk for calcium deficiency? An increased intake of fast foods Iron deficiency The adolescent growth spurt Drinking less milk

Drinking less milk

A student in a nutrition class is giving a report on "Adolescent Obesity." One of the topics in the report is health risks increased by obesity. What health risk would you expect to be included in the report? Type 1 diabetes mellitus Hypercalcemia Dyslipidemia Hypercholesterolemia

Dyslipidemia

Acute episodes of illness can create problems for clients who have insulin-dependent diabetes. During these periods of acute illness, what should clients who have insulin-dependent diabetes do? 1. Avoid taking any insulin or oral diabetic medications Consume the exact amount of normal carbohydrate allowance Avoid drinking fruit juices or fruit purees Eat about three carbohydrate choices every 4 hours

Eat about three carbohydrate choices every 4 hours

The school nurse is providing nutrition education to parents of adolescents. Which of the following foods should the nurse include in suggestions to enhance nutritional choices for this age group? Eat dinner together as a family. Skip breakfast for a bigger lunch. Eat out at a restaurant as a family. Eat carbohydrate-dense snacks.

Eat dinner together as a family.

The nurse is conducting a class for a group of clients with newly diagnosed type 2 diabetes mellitus. Which of the following are suggestions to help the clients make better decisions? Select all that apply. Eat out less often. Bring healthy snacks from home. Keep a log of foods eaten daily. Stock up on "fat-free" and "sugar-free" nutrition bars. Switch to whole milk.

Eat out less often. Bring healthy snacks from home. Keep a log of foods eaten daily.

A client with malabsorption syndrome is experiencing anorexia. Which of the following dietary interventions should the nurse recommend to increase intake? Avoid milk products. Eat a low fat diet. Eat six small meals every two hours. Increase fluid intake.

Eat six small meals every two hours.

The nurse is seeing a client in the prenatal clinic who reports feeling as though food is coming back up from her stomach. She reports belching often after a meal. Which of the following interventions should the nurse recommend? Eat small, frequent meals. Drink liquids immediately before meals. Eat easily digested carbohydrates. Drink warm liquids with the meals.

Eat small, frequent meals.

A family member is concerned that an older adult family member does not have adequate nutritional intake. Which of the following is a risk factor for malnutrition among the older adult population? Having home health assistance three times a week to fix meals Traveling to a local center for a hot meal Eating alone Daily Meals on Wheels delivery

Eating alone

The nurse is providing teaching to a client being discharged to finish recovering from a small bowel resection and ileostomy. The client is concerned about what the best diet is for him. What would be the best response? A regular diet with no restrictions Eating small, frequent meals Increase fat intake for added calories Large meals to produce large amounts of stool

Eating small, frequent meals

To assist in nutrition screening in the community, the local older adult center has developed a screen to help identify individuals at high risk for malnutrition. Which of the following risk factors might be included? 1. Muscle mass loss 2. Sudden weight loss 3. Eats alone most of the time 4. Body mass index

Eats alone most of the time

Unintentional weight loss can be an indicator of malnutrition; however, apparent weight gain may be an unreliable indicator of improvement due to which of the following? 1. Carcinoma 2. Edema 3. Cholecystitis 4. Pancreatitis

Edema

The nurse is caring for a 65-year-old Mr. Y who was admitted 1 week ago after his family found him confused and unable to ambulate in his home. After conducting the Mini Nutritional Assessment, the nurse notes the client has a score of 13 points. Which of the following client problems will the nurse find? 1. Stable mood 2. Edema of the feet 3. Sudden weight loss 4. Increased fat mass

Edema of the feet

A 47-year-old, obese female client is admitted for hypertensive crisis. During the assessment, she states she has tried every diet out there and she is not interested in trying another one. Which of the following is the best approach for the nurse to use to help this client? 1. Report the client to the physician and note it in her medical record. 2. Emphasize things "to do" instead of "not to do." 3. Ask a social worker to intervene. 4. Provide the client with colorful graphs and charts to note the foods she eats.

Emphasize things "to do" instead of "not to do."

When teaching a client about diet changes, the nurse knows it is better to emphasize which of the following? 1. Encourage calorie counting daily 2. Emphasize low fats and carbohydrates 3. Encourage healthy options often 4. Emphasize avoiding bad foods

Encourage healthy options often

A client who was widowed 6 weeks ago has a decreased appetite since his wife died. He reports consuming a low volume of food and worsening appetite. He has unintentionally lost 19 pounds since his wife's death and reports around-the-clock fatigue. He is admitted to the unit for weight loss due to inadequate nutritional intake. What nursing intervention would be appropriate for the nurse to include in the care plan for this client? 1. Suggest that the client eat alone in his room. 2. Encourage the client to eat a large, dense breakfast. 3. Instruct the client to avoid snacking in between meals. 4. Ensure the client orders a wide variety of foods to increase volume of intake.

Encourage the client to eat a large, dense breakfast.

Nutrition therapy in the elderly has different goals than at other times across the lifespan. What is a primary goal of diet intervention for the elderly? Prevent the development of complications Enhance quality of life Treat chronic disease Prolong life

Enhance quality of life

The nurse is assessing a 39-year-old woman's diet who has been experiencing chronic constipation. When recommending nonpharmacologic interventions, which of the following should the nurse include? Adding more fruits and vegetables alone will help. Ensure at least eight glasses of water are consumed daily. Get plenty of rest during the day time hours. Increase dietary fiber intake to 38 g/day.

Ensure at least eight glasses of water are consumed daily.

The nurse is admitting a client with burns over 60% of the body. The nurse should prepare for which of the following nutritional interventions for this client? Parenteral nutrition Oral nutrition with a liquid supplement Enteral nutrition Oral nutrition

Enteral nutrition

A client is at the clinic for a routine checkup. The nurse is asked how much time would have to be devoted to an exercise program to lose some weight. Which of the following is the best response to this client's question? Just increase the intensity of the activity the client is already doing. Increase energy expenditure by 2500 or more calories per week. Reduce daily calorie intake by 350 cal/day to lose a pound a week. Exercise at least 300 minutes/week and adjust diet.

Exercise at least 300 minutes/week and adjust diet.

The nurse is admitting a 42-year-old client with colon cancer to the unit. The client is scheduled to have a resection with ileostomy. The nurse should include which of the following information in preoperative teaching? Select all that apply. Extra protein and calories will need to be incorporated into the diet. Fluid intake will need to be limited to 4 to 5 cups per day after the surgery. A low-fiber diet will eventually help with reduction of stool output. Clear liquids with simple sugars will be permitted just after the surgery. If foods are not tolerated initially they should be avoided indefinitely.

Extra protein and calories will need to be incorporated into the diet. A low-fiber diet will eventually help with reduction of stool output. Clear liquids with simple sugars will be permitted just after the surgery.

The nurse is conducting client education with a group of clients newly diagnosed with hypertension. The nurse assesses the effectiveness of the education by questioning how healthy sodium balance is maintained in the body. Which of the following is a correct response? Extra sodium leaves the body through feces. Extra sodium is absorbed into body tissues. Extra sodium leaves the body through urine. Extra sodium is deposited on bones.

Extra sodium leaves the body through urine.

A student in the nutrition class is giving a report on "Promoting Exercise Adherence in Obese Clients." What strategy would be expected to be included in the report? Walking with a buddy at the mall promotes socialization as well. A gym provides the structure needed to exercise effectively. Find creative ways to work out at home rather than the gym. At least 45-minute session of exercise per day is required for it to be effective.

Find creative ways to work out at home rather than the gym.

Children sometimes get mottled tooth enamel because they ingest more fluoride than is recommended while their teeth are forming. Which of the following may be the cause of this? Fluoride in foods Fluoride supplements Drinking water Fluoridated toothpaste

Fluoridated toothpaste

Given that bottled water is highly consumed by Americans, the nurse assessing the health needs of children in the community is aware that children who drink bottled water may require supplementation with which of the following? Fluoride Iron Magnesium Chlorine

Fluoride

The nurse is conducting a nutritional screening with an 84-year-old client with depression. Further into the assessment, the client describes positive feelings associated with certain foods eaten with his family in the past. In planning his meals, what would be appropriate for the nurse to include? 1. Foods that are considered "comfort foods" 2. Favorite "takeout" food 3. Foods your client tolerates well 4. Foods high in nutritional value

Foods that are considered "comfort foods"

While conducting a class on homeostasis as a function of the body, which of the following statements about homeostasis and minerals is true? For some minerals, the body maintains homeostasis by releasing minerals from storage for redistribution. For some minerals, the body maintains homeostasis by altering the mineral excreted. For some minerals, the body maintains homeostasis by using one mineral in place of another. For some minerals, the body maintains homeostasis by altering the mineral absorbed.

For some minerals, the body maintains homeostasis by releasing minerals from storage for redistribution.

The nurse is meeting with the wife of a client who has Alzheimer disease (AD). She is concerned about her husbands' nutritional status. What can the nurse tell her to watch for that would indicate a possible nutritional problem? Select all that apply. Forgetting what time to eat Forgetting which are green vegetables Forgetting he had eaten and is eating again Forgetting how to shop for food Forgetting what food category bread is in

Forgetting he had eaten and is eating again Forgetting how to shop for food

Parents of 4-month-old baby ask what concerns they should have about formula feeding. How should the nurse respond? Formula-fed babies are at risk for being overfed. You may risk using the wrong formula for your baby. Dehydration can be common in babies who are formula fed. Formula-fed babies can consume too much iron.

Formula-fed babies are at risk for being overfed.

The nurse is conducting client teaching for a new mother who is formula feeding her infant. The client asks about the nutritional value of a formula. What would be the nurse's best response? Formulas made in the United States have lower nutritional values than are contained in breast milk. Formulas made in the United States have minimum nutrient requirements that are more than the nutrient amount provided in breast milk. Formulas made in the United States have the same nutrient value as breast milk. Formulas made in the United States need to be supplemented with iron.

Formulas made in the United States have minimum nutrient requirements that are more than the nutrient amount provided in breast milk.

To counteract low serum levels of vitamin B12 typically found in older adults, the nurse should recommend and increase in intake which of the following sources of this vitamin? Fortified cereals Red meats Citrus fruits and juices Milk and dairy products

Fortified cereals

The nurse is seeing a 58-year-old client who would like to know what to eat in order to increase intake of vitamin D. Which of the following responses by the nurse correct? Fortified milk Green leafy vegetables Nuts Lean red meats

Fortified milk

The nurse is teaching a nutrition class at the local high school. When discussing good sources of dietary calcium, which of the following foods should the nurse recommend? Egg yolks Potatoes Fortified orange juice Organ meats

Fortified orange juice

A client with diabetes is meal planning using the food list for diabetes. According to the food list, a serving of milk may be exchanged for which of the following? Select all that apply. Vegetables Meat Fruit Other carbohydrates Starch

Fruit Starch

The nurse is caring for a 33-year-old female who was in a motor vehicle accident. She is in the stress response phase. Which amino acid is depleted rapidly during stress? Glycine Glutamine Alanine

Glutamine

The nurse emphasizes that nutrition therapy is the basis of treatment for all clients with diabetes in the nutrition class. Which of the following nutrition recommendations for clients with diabetes is true? Together, total carbohydrate and monounsaturated fat should provide less than 50% of total calories. Gram for gram, sucrose cannot replace starch without causing a deleterious effect on glucose levels. Glycemic control is dependent on matching carbohydrate intake with the action of insulin or other medication. Natural sources of fructose (fruits, vegetables) should be restricted.

Glycemic control is dependent on matching carbohydrate intake with the action of insulin or other medication.

The nurse is seeing 19-year-old who shows signs of prediabetes. The client wants to know what strategies can be used to prevent the onset of type 2 diabetes. Which of the following should the nurse recommend? Use a 1200-calorie plate method plan. Lose 15% of current body weight. Go for a brisk walk 30 minutes per day. Opt for a carbohydrate-free diet.

Go for a brisk walk 30 minutes per day.

When providing teaching on feeding hazards with preschoolers, the nurse should include which of the following foods when discussing the risk for choking? Noodles Soda crackers Cubes of cheese Grapes

Grapes

The nurse is conducting an assessment of the 38-year-old male who is requesting assistance with weight loss. The nurse determines the client has abdominal obesity based on the waist circumference of which of the following? Greater than 35 in Greater than 45 in Greater than 30 in Greater than 40 in

Greater than 40 in

A nurse is seeing a client in a weight loss clinic who asks what she can add to her diet to help her feel fuller and better able to stick to a low-calorie regimen. Which of the following foods would be best for the nurse to recommend? Whole wheat bread Grilled chicken breast Whole milk Bananas

Grilled chicken breast

The nurse can anticipate which of the following eating strategies may be effective for clients with bulimia nervosa? Have a meal plan at the start of each day. Use finger foods. Avoid "forbidden" or problem foods. Eat with distractions.

Have a meal plan at the start of each day.

The son of a 90-year-old client who is sedentary is discussing with a nurse strategies for preventing malnutrition in the older adult. How should the nurse best respond? Puree foods the client finds enjoyable. Ensure the client is consuming only heart-healthy food. Provide a quiet environment for the client during meal times. Have some family meals throughout the week.

Have some family meals throughout the week.

A nurse working on a very busy unit feels she does not have adequate time to monitor each client's nutritional intake. What is a simple nursing intervention that can assist the nurse in evaluating clients' intended outcomes? 1. Record the client's output and compare with intake. 2. Encourage the client to document what is ordered at each meal. 3. Have the client weighted daily and record the weight. 4. Have the client's family record the amount of food served at each meal.

Have the client weighted daily and record the weight

If a client is overweight or obese, setting the goal of reaching the "ideal" weight may be very hard to accomplish. To achieve health benefits from weight loss, how much weight does an individual actually have to lose? Health benefits cannot be measured. Health benefits do not always occur, even if weight loss is significant, if obesity is chronic. Health benefits can be realized with only a modest weight loss. Health benefits occur only when ideal body weight is achieved.

Health benefits can be realized with only a modest weight loss.

Osteoarthritis (OA) is the most common form of arthritis. What is the greatest known modifiable risk factor of OA of the knee? High body mass index Inadequate intakes of calcium Moderately inactive lifestyle Nutritional deficiency of folic acid

High body mass index

An 80-year-old female was admitted with pneumonia. During her assessment, she mentions she has recently had difficulty eating because "everything keeps going down the wrong pipe." When the nurse is developing a care plan, which of the following is an appropriate nursing diagnosis to include? 1. High risk of aspiration 2. Rule out aphasia 3. Increased mucus production 4. COPD

High risk of aspiration

The nurse is conducting nutritional screening for older adults at a health fair. Which of the following would be an important question to ask while screening an older adult for nutritional risk? Who lives in your home? Are you over the age of 62 years? Have you lost more than 3% of your body weight in the past 2 months? How many medications do you take?

How many medications do you take?

The nurse is working with clients at the clinic develop appropriate weight loss goals. The nurse is aware that people who store a disproportionate amount of body fat in the abdomen are at increased risk for what? Type 1 diabetes mellitus Hypertension Respiratory diseases Hypolipidemia

Hypertension

Nursing care plans include nursing diagnoses that address nutrition status when the nurse assesses this as a client need. After identifying a nutritional deficiency in a client, which nursing diagnosis would be most appropriate to include? 1. Impaired oral mucous membrane 2. Deficient fluid volume: less than body requirements 3. Constipation 4. Imbalanced nutrition: less than body requirements

Imbalanced nutrition: less than body requirements

The nurse is admitting an 18-year-old male to the ICU following a motor vehicle accident. The client is currently receiving IV fluids with no nutrition until the client is stabilized. When should this client start on nutritional support? In 36 to 48 hours In 12 to 24 hours Immediately In 24 to 48 hours

In 24 to 48 hours

During a clinic visit, the mother of 3-month-old infant tells the nurse the baby is taking Phenyl-free formula. For which of the following reasons would the infant require this type of formula? Gastroesophageal reflux Constipation Cystic fibrosis Inborn error of metabolism

Inborn error of metabolism

A 19-year-old college student has come to the college health clinic for information on how to decrease his risk for developing diabetes. He is concerned because he is overweight and he just learned his mother has been diagnosed with type 2 diabetes. Which of the following suggestions should the nurse offer this client? Exercise 200 minutes per week at high intensity. Increase consumption of nuts and berries. Weight loss of at least 15% within the next 6 months. Eliminate fats from his diet.

Increase consumption of nuts and berries.

A client has been instructed to start a high-fiber diet to treat chronic constipation. Which of the following suggestions will help the client make the transition easier? Increase the fiber intake gradually. Add 1 cup of bran a day immediately. Add lots of nuts and seeds to the diet. Always peel any fruit before eating.

Increase the fiber intake gradually.

The nurse is aware that making the decision to lower calorie intake requires more than just eating less. Which of the following recommendations should the nurse make to help clients meet nutritional needs without exceeding caloric limits? Increasing sugar-free foods Increasing complex carbohydrate intake Increasing nutrition density Increasing fat-free foods

Increasing nutrition density

The nurse is evaluating the client's nutritional status at her prenatal visit. The client is asking if she should take a nutrient supplement. The nurse should base the response to the client's question on which of the following needs? Increased nutrient needs during pregnancy The stress of pregnancy A decrease in eyesight Individual circumstances

Individual circumstances

The nurse is discussing meal planning with an adolescent client who has type 1 diabetes and the client's mother. Which of the following plans should the nurse suggest for this client? High glycemic index Carbohydrate counting Individualized meal plans Exchange lists

Individualized meal plans

The nurse is teaching a nutrition class for nursing students. The discussion for today is about the nutrition therapy for malabsorption disorders. The students correctly state which of the following interventions can help manage symptoms of this disorder? A low-fat, high-fiber diet for all malabsorption disorders Clear liquids with vitamin and mineral supplements Small, frequent meals that are low in soluble fiber Individualized therapy according to symptoms and complications

Individualized therapy according to symptoms and complications

Which of the following foods would be the best option for introducing to a 6-month-old infant? Infant rice cereal Strained infant fruits Non-citrus fruit juices Infant vegetables

Infant rice cereal

The nurse is working with a client who was recently been found to have lactose malabsorption. The client needs to adapt to a low-lactose diet. Which of the following should the client avoid? Rice milk Yogurt Soy sour cream Instant soups

Instant soups

The nurse conducts an educational session on nutrition for staff in a long-term care facility. The nurse asks the learners to complete an assignment to create a list of strategies for enhancing food intake in residents of the facility. If teaching was effective, which of the following strategies should be on this list? Select all that apply. Let residents eat in their rooms. Involve the family in mealtime. Honor food preferences. Encourage independence in eating. Make sure the diet is heart healthy.

Involve the family in mealtime. Honor food preferences. Encourage independence in eating.

The nurse is discussing with the nutrition class the role that some nutrients play in regulating chemical substances in the body. The nurse correctly explains which of the following is essential for effective metabolism and thermoregulation? Iodine Zinc Iron Selenium

Iodine

When family members of a client who is receiving trophic enteral feeding ask why the client is being underfed, which of the following responses should the nurse provide? It promotes a hyperglycemic response in the body. It increases the inflammatory response in the body. It is better than providing the client with a full calorie target diet. It decreases the amount of metabolic waste the body produces.

It decreases the amount of metabolic waste the body produces.

The nurse is conducting a nutrition class when one of the clients questions the accuracy of using BMI to determine obesity. The client is surprised to see his BMI is high but he does not feel he is overweight. What is a drawback of using body mass index to determine obesity? It does not take diet into account. It is nonthreatening. It does not take body composition into account. It requires special equipment to measure.

It does not take body composition into account.

The nurse is caring for a client for whom serum albumin screening has been used to assess protein status. The nurse understands that there are disadvantages to using albumin to indicate the nutritional status of protein in the body. What is one of those disadvantages? 1. It is not specific for nutritional status. 2. It is a test that requires 12 hours of fasting. 3. It must be assessed in relation to the client's blood urea nitrogen. 4. It is degraded very quickly.

It is not specific for nutritional status.

Refeeding syndrome is a potentially fatal complication of overfeeding. For which of the following reasons should the nurse anticipate the signs of refeeding in clients receiving nutrition therapy? It occurs as a result of a low-calorie diet. It may lead to hypovolemic shock. It results in a decrease in insulin. It may result in dangerous shifts in serum electrolytes.

It may result in dangerous shifts in serum electrolytes.

The nurse is teaching a nutrition class with a group of student nurses. The nurse knows teaching has been effective when students indicate which of the following strategies should be employed for people with prediabetes? Keeping track of weight on a weekly basis Avoiding culturally preferred foods Avoiding use of meal replacements Choosing vegetables only

Keeping track of weight on a weekly basis

The nurse is seeing an older adult client who states her health-care provider informed her she is not getting enough magnesium in her diet. Which of the following foods should the nurse recommend to help the client obtain more of this nutrient? Potatoes Fruit juices Leafy greens Carrots

Leafy greens

A client calls the clinic complaining of diarrhea that has lasted 36 hours. The nurse recommends which of the following nutrition interventions for this client? Low-potassium foods Liberal intake of apple juice to replace nutrients Liberal fluid intake Clear liquids only to decrease bowel stimulation

Liberal fluid intake

The nurse is conducting client education with a client newly diagnosed with diabetes. Which of the following are appropriate initial treatments for mild hypoglycemia? Select all that apply. Chocolate cookies Life Savers A chocolate bar A soft drink Gum drops

Life Savers A soft drink Gum drops

A nurse prepares to share several suggestions learned from a conference titled "How to Facilitate Client and Family Nutritional Teaching" with other unit nurses. Which of the following is an appropriate suggestion for teaching clients and their families? 1. Listen to the ideas and concerns the client and family may have. 2. Suggest books to both the client and his or her family to read. 3. Emphasize to the client's family foods to be avoided. 4. Make sure the client selects lots of "comfort foods."

Listen to the ideas and concerns the client and family may have.

The nurse is preparing a meal plan for a client who will be assisted by a home health aide. The nurse recognizes the overall goal of nutrition therapy for this older adult is which of the following? Maintain or restore maximal independent functioning Decrease the use of medication Prevent chronic disease Reduce chronic disease deterioration

Maintain or restore maximal independent functioning

Evidence suggests that enteral nutrition (EN) is superior to parenteral nutrition (PN) and will impact client mortality among the critically ill in the ICU. Early EN has been noted to do which of the following? Select all that apply. Prevent an ileus Maintain the integrity of the gastrointestinal mucosa Prevent weight loss Modulate inflammatory responses

Maintain the integrity of the gastrointestinal mucosa Modulate inflammatory responses

A nurse is discussing strategies to encourage healthy eating behaviors in children. Which of the following points would be important to include? Select all that apply. Provide television as a distraction. Make sure snacks are nutrient-dense. Encourage physical activity throughout the day. Parents should sit down to eat with their children. Offer one food group at a time.

Make sure snacks are nutrient-dense. Encourage physical activity throughout the day. Parents should sit down to eat with their children.

A nurse is discussing strategies to encourage healthy eating behaviors in children. Which of the following points would be important to include? Select all that apply. Make sure snacks are nutrient-dense. Offer one food group at a time. Parents should sit down to eat with their children. Encourage physical activity throughout the day. Provide television as a distraction.

Make sure snacks are nutrient-dense. Parents should sit down to eat with their children. Encourage physical activity throughout the day.

The client has come to the office with an exacerbation of his celiac disease. He says he has been reading the labels and making sure he avoids wheat, barley, and rye. After reviewing the labels with the client, the nurse recognizes which listed ingredient is responsible for the client's symptoms? Millet Carrageenan Malt flavoring Sorghum

Malt flavoring

The nurse is helping a 76-year-old woman develop an exercise program. Which of the following are important to consider? Select all that apply. Match her activities to her abilities. Include exercises to improve balance. Work out longer but slower to improve her activity level. Work up to 150 minutes of moderate-intensity aerobic activity per week. Determine her level of effort.

Match her activities to her abilities. Include exercises to improve balance. Determine her level of effort.

The nurse is teaching a high school health class about calcium. What should the nurse teach the students to eat in order to promote the absorption of calcium? Milk Beef Coconut oil Bran cereal

Milk

Nutrient deficiencies in young children are generally not a health risk. However, young children who drink too much milk are at risk for which of the following issues? Protein deficiency Hypervitaminosis A Milk anemia Hypercalcemia

Milk anemia

A nurse educator is teaching a class about the role of minerals in the diet. The nurse is assesses that teaching was effective when students are able to verbalize which of the following? Minerals are broken down and rearranged during metabolism. Minerals originate from the earth's crust. Minerals are digested primarily in the small intestine. Minerals are easily destroyed by light, air, heat, and acids.

Minerals originate from the earth's crust.

Research regarding increasing the effectiveness of client teaching has found that nutritional counseling that is most effective is that done by which health-care provider? 1. Nurses 2. Dietitians 3. Physicians 4. Nurses and dietitians

Nurses and dietitians

The client has a 6-month-old infant. It is part of the infant's bedtime routine to get a bottle of formula when he goes down for the night. Which of the following problems is this infant most likely at risk for? Consuming too many calories Gastric reflux Developing diabetes later in life Nursing bottle caries

Nursing bottle caries

The nurse is assessing the client's BMI during a routine physical examination. The client is a 19-year-old male who stands 6 ft tall and weighs 235 pounds. The nurse determines the client falls into which BMI category? Obesity class 1 Healthy weight Overweight Obesity class 2

Obesity class 1

A client is found to have a BMI of 37. This indicates the client is in which weight category? Obesity class 1 Obesity class 4 Obesity class 2 Obesity class 3

Obesity class 2

The client is a 38-year-old male who is critically ill. The client's appetite remains low. He is afraid of gaining too much weight. Which of the following would be an example of an appropriate meal for this client? 2 strips bacon, 2 scrambled eggs fried in butter, 1 slice sourdough bread, and coffee Granola with honey, fresh strawberries and milk, and hot tea with honey Omelet with ham, cheese, mushroom, and spinach; whole wheat toast with butter and jelly; and milk and coffee with whipped cream Smoothie made with yogurt, instant breakfast mix, and skim milk and peaches

Omelet with ham, cheese, mushroom, and spinach; whole wheat toast with butter and jelly; and milk and coffee with whipped cream

The nurse is aware that there will be times when enteral nutrition (EN) will not be feasible to use with a critically ill client. Which of the following are indications to use parenteral nutrition (PN)? Select all that apply. PN should be initiated only if the anticipated length of therapy is 4 days or more. On the 8th hospital day in a well-nourished client who has received nutrition For 3 to 4 days preoperatively for clients expected to undergo major reconstructive surgery As soon as possible in malnourished clients

On the 8th hospital day in a well-nourished client who has received nutrition As soon as possible in malnourished clients

The nurse is aware that there will be times when enteral nutrition (EN) will not be feasible to with a critically ill client. Which of the following are indications to use parenteral nutrition (PN)? Select all that apply. On the 8th hospital day in a well-nourished client who has received nutrition As soon as possible in malnourished clients For 3 to 4 days preoperatively for clients expected to undergo major reconstructive surgery PN should be initiated only if the anticipated length of therapy is 4 days or more.

On the 8th hospital day in a well-nourished client who has received nutrition As soon as possible in malnourished clients

The nurse is reviewing the nutrition orders for a newly admitted client with pneumonia. The client requires restriction of fluids. The nurse anticipates what type of enteral feed will be ordered for this client? One that provides 250 calories per 250 ml One that provides 150 calories per 250 ml One that provides 500 calories per 250 ml One that provides 750 calories per 250 ml

One that provides 500 calories per 250 ml

The nurse assessing a client with lactose intolerance is likely to find the client is having which of the following type of bowel movements? Osmotic diarrhea Secretory diarrhea Watery No diarrhea

Osmotic diarrhea

A person's body mass index (BMI) provides data regarding the relative level of health associated with a person's weight range. Someone with a BMI of 25 would be considered which of the following? Underweight Obese Of normal weight Overweight

Overweight

A 6-year-old boy has just come in from school and wants a snack. The mother is trying to maintain a heart-healthy diet for him by following the Dietary Guidelines for Americans. What would be the best snack the mother could give him? Hide answer choices Ice cream Peanut butter on crackers Cheese-flavored popcorn Low-salt potato chips

Peanut butter on crackers

The nurse is seeing a client who is 65 years old. The client makes the statement, "Why should I care so much about what I eat, I won't be living that much longer anyway." Which of the following is true about aging? It is important to consider end-of-life issues at this age. People are living much longer than before. There is little quality of life for people past 55 years. In older adulthood, health quality is the same for all.

People are living much longer than before.

A nursing student is taking a course in geriatric nursing, and the subject for today is altered nutritional status. Which of the following data from the physical assessment is least reliable in indicating to the nurse that the older adult client might be malnourished? 1. Inability to talk normally 2. Poor wound healing 3. Edematous hands and forearms 4. Enlarged liver with nonpalpable spleen

Poor wound healing

When preparing resources to help clients understand portion control, which of the following widely used terms should the nurse include to describe the American phenomenon of "super-sized" portions? Increased appetite Portion identity Increased hunger Portion Distortion

Portion Distortion

A nurse is providing teaching to a client who has recently started consuming a lacto-vegetarian diet. The nurse is correct in informing the client that she will be able to take in more of which mineral? Sodium Potassium Iron Iodine

Potassium

The elderly population has a mean intake below the dietary reference intake (DRI) for several nutrients. Which of the following falls into that classification? Manganese Potassium Chloride Sodium

Potassium

Quality-of-life issues are a priority in the elderly, especially in residents of long-term care facilities. What are the key components in meeting quality-of-life standards among older adults who are residents of long-term care facilities? Prevention of unintentional weight loss and pressure ulcers Prevention of weight gain and hyperlipidemia Prevention of depression and diabetes Prevention of low albumin and anorexia

Prevention of unintentional weight loss and pressure ulcers

While teaching a nutrition class, the nurse correctly discusses the largest source of sodium in the typical American diet is which of the following? Salt added during cooking Processed foods Salt added at the table Foods naturally high in salt

Processed foods

A client is trying to understand where cheese fits into the MyPlate for Older Adults. The nurse explains that according to this plan, cheese is in which section? Dairy Protein Fat Milk

Protein

The 74-year-old client who has been hospitalized for 1 week has developed a pressure ulcer on her sacrum that has become infected. Which of the following reasons is the most likely cause of this health complication? Protein-calorie malnutrition Obesity Lack of vitamin B Excessive movement in bed

Protein-calorie malnutrition

The client is being discharged home on a high-protein diet. The client asks the nurse what would be the best dessert to help increase intake of protein. Which of the following is the nurse's best response? Sherbet Gelatin Cherry pie Pudding

Pudding

While teaching about the Recommended Dietary Allowances (RDAs), the nurse has included which of the following points in the discussion? 1. RDAs are the same as dietary reference intakes. 2. RDAs represent requirements for macronutrients only. 3. RDAs recommendations apply to people with chronic diseases. 4. RDAs are appropriate for healthy individuals in all age groups.

RDAs are appropriate for healthy individuals in all age groups.

Numerous health agencies in the United States have issued dietary recommendations so that Americans can choose diets that are aimed primarily at which of the following goals? 1. Reducing the risk of acute heart disease 2. Reducing their weight 3. Reducing their risk of chronic disease 4. Increasing their activity levels

Reducing their risk of chronic disease

The client is interested in starting an exercise program that will provide the most benefit. What type of exercise is known to raise the metabolic rate? Flexibility exercise Resistance exercise Intense aerobic exercise Low-impact aerobic exercise

Resistance exercise

The nurse is caring for a client with an acute lung injury from trauma who is transitioned to an oral diet. The client reports dry mouth, constipation, and fatigue. Which of the following suggestions can the nurse make to help improve this client's status? Select all that apply. Drink coffee with his meal. Rest before eating. Increase fiber intake. Use the bronchodilator after eating. Add gravies and sauces to food

Rest before eating. Increase fiber intake. Add gravies and sauces to food

The nurse educator is teaching a group of nursing students about primary and secondary prevention of diabetes. The nurse knows teaching has been effective when students correctly state people with diabetes urged to limit their intake of which of the following? Nonnutritive sweeteners Sucrose Saturated fat Fiber

Saturated fat

The nurse educator is teaching a group of nursing students about primary and secondary prevention of diabetes. The nurse knows teaching has been effective when students correctly state people with diabetes urged to limit their intake of which of the following? Sucrose Fiber Saturated fat Nonnutritive sweeteners

Saturated fat

Antioxidants disarm free radicals produced during normal oxygen metabolism. Which mineral is a component of an enzyme that acts as an antioxidant? Zinc Iron Selenium Iodine

Selenium

A 77-year-old client who lives alone is admitted to the hospital after his children found him in a confused state at home. As the nurse is preparing the nursing care plan, which of the following nursing diagnoses would indicate a nutrition intervention is required? 1. Self-care deficit: feeding 2. Risk for activity intolerance 3. Risk for impaired skin integrity: colostomy 4. Risk for impaired swallowing: resolved

Self-care deficit: feeding

During a class on fluid balance, the nurse educator discusses water as a vital element for the human body. The class has effectively learned at least one of the functions of water when they state which of the following? Makes cell walls without shape and structure Provides energy Prepares nutrients for transportation to cells Serves as a solvent for vitamins and glucose

Serves as a solvent for vitamins and glucose

The nurse is conducting nutrition counseling with a 21-year-old pregnant woman in a prenatal clinic. What should the nurse explain to the client about caloric intake during the second and third trimesters of her pregnancy? She will need approximately 500 cal/day increase during the third trimester. She will need approximately 500 cal/day increase during the second trimester. She will need approximately 450 cal/day increase during the second trimester. She will need approximately 450 cal/day increase during the third trimester.

She will need approximately 450 cal/day increase during the third trimester.

The nurse is counseling a clinic client who has short bowel syndrome on how to change his diet from regular to low fat. What would be the best food to include in the menu plan? Sherbet Coconut Avocado Biscuits

Sherbet

A client is admitted with persistent diarrhea and a weight loss of 10 pounds. Which of the following assessment data is significant about the weight loss if the client stated it happened over the past 3 months? 1. Signifies there is no nutritional deficit 2. Signifies a chronic versus acute condition 3. Signifies that it was an intentional weight loss 4. Signifies an ongoing acute condition

Signifies a chronic versus acute condition

When preparing information about contemporary problems with healthy eating in children, which of the following issues should the nurse include? Select all that apply. Skip breakfast. Obtain a significant portion of their calories from sweetened beverages. Get one third of their calories from snacks. Eat at least two heart-healthy meals per day. Eat less protein than their needs.

Skip breakfast. Obtain a significant portion of their calories from sweetened beverages. Get one-third of their calories from snacks.

The school nurse is seeing a 14-year-old girl who says she does not feel like eating breakfast in the morning. The nurse should provide which of the following information about skipping breakfast to help the adolescent make healthier decisions for her daily nutrition intake? Skipping breakfast is linked with becoming obese in adulthood. Skipping breakfast will lead to having high cholesterol levels in adulthood. Skipping breakfast can make it more difficult to concentrate and retain information. Skipping breakfast is something most teens stop doing if they play sports.

Skipping breakfast can make it more difficult to concentrate and retain information.

The nurse is admitting to the unit a client who is a 42-year-old female with acute respiratory distress syndrome (ARDS). While completing the admission assessment, the nurse finds out that this client is being treated for depression and has had little or no appetite for over a month. The nurse should include which of the following interventions in the client nutritional plan? High-calorie, high-protein meals three times a day Small frequent meals spread out over the day Regular diet with vitamin and mineral supplements with the meals High-fiber diet with enteral supplements between the three meals

Small frequent meals spread out over the day

The client is complaining that she is tired of the commercial high-calorie, high-protein drink and would like to have something different. Which of the following would be a good substitute for a snack? Cup of mixed fruit Smoothie made with yogurt, instant breakfast mix, whole milk, and strawberries Snack crackers with cheese and grapes Milkshake made with blueberries, frozen yogurt, and skim milk

Smoothie made with yogurt, instant breakfast mix, whole milk, and strawberries

The nurse is admitting a client who has developed dehydration secondary to diarrhea. After treatment to rehydrate the client, which of the following blood levels should have returned to within the normal range? Select all that apply. Phosphorus Calcium Sodium Potassium

Sodium Potassium

should be sure to discuss allergies are more likely with which one of the following foods? Tomatoes Corn oil Soy Rice

Soy

The nurse is speaking to an exercise class when one of the participants asks, "Why should I do strength training?" Which of the following would be the best response to this question? Strength training gives you a really good body. Strength training decreases the metabolic rate. In strength training, warm-up and cool-down periods are not required. Strength training improves bone density.

Strength training improves bone density.

The nurse is seeing an older adult client who has been assessed to have sarcopenia. The nurse is providing education to the client regarding lifestyle modifications that can be made to halt or reverse this condition. Which of the following modifications should the nurse recommend? Strength-training exercises Aerobic exercise A high-protein diet A high-protein diet, high-calorie diet

Strength-training exercises

The client is being evaluated for dehydration. The nurse is aware which of the following conditions increase the loss of fluid from the body? Thirst Congestive heart failure Controlled diabetes Strenuous exercise

Strenuous exercise

The nurse is caring for a client with a severe protein deficiency. What other deficiency would the nurse anticipate to find in this client? Chromium Molybdenum Selenium Sulfur

Sulfur

The nurse has been emphasizing the importance of reading labels to know the content of foods the client is eating. When reading a nutrition label, the client recognizes which of the following fats is the best option? 1. Partially hydrogenated oil 2. Milk fat 3. Palm oil 4. Sunflower oil

Sunflower oil

It is well recognized that most Americans have become more sedentary than in the past. A sedentary lifestyle increases the risk for various diseases and complications. Experimental studies have shown that prolonged sitting can lead to which of the following? Suppression of insulin activity in the muscles An increased use of stored fat for energy A decreased risk for osteoporosis An increase in the discomfort related to menopause

Suppression of insulin activity in the muscles

The nurse is assisting a client with celiac disease to plan a daily menu. What food would be appropriate for this client? White bread Instant cocoa Tapioca pudding Wild rice

Tapioca pudding

The client has not been eating well since he was admitted to the long-term care facility. When the advanced practice nurse (APN) comes to the facility to see her clients, the nurse mentions her concerns about the nutritional status of this client. The APN notes that one of the client's orders is for a can of liquid dietary supplement to be given between meals three times a day. What may the client be experiencing as a result of taking this supplement? Select all that apply. Increased hunger Taste fatigue Increased sugar and fat Decreased calcium Loss of appetite

Taste fatigue Loss of appetite

Part of the Dietary Reference Intakes is the Tolerable Upper Intake Level (UL). Which of the following would be the correct explanation for the nurse to provide the client regarding these amounts? 1. The UL is the estimated amount of nutrient that will meet the needs of 90% of a given population. 2. The UL is the recommended level of intake for that nutrient. 3. The UL is the nutrient intake amount determined to be most beneficial for preventing chronic disease. 4. The UL is the highest level of daily nutrient intake that is likely to pose no risk of adverse health effects.

The UL is the highest level of daily nutrient intake that is likely to pose no risk of adverse health effects.

The nurse correctly explains to a group of older adults that which of the following causes the body's need for vitamin D to increase over the age of 50 years? The increased consumption of coffee and tea interferes with vitamin D synthesis The potential risk for coronary heart disease, which uses vitamin D The ability to absorb calcium, which decreases with age The ability to synthesize vitamin D from sunlight, which decreases with age

The ability to synthesize vitamin D from sunlight, which decreases with age

The nurse is discussing metabolic stress and how it creates protein catabolism with the nutrition class. The nurse knows teaching has been effective when students correctly state which of the following are indications of protein catabolism occurring? The body goes into a negative nitrogen balance. The body is full of free fatty acids. The body is full of free radicals. The body goes into a positive nitrogen balance.

The body goes into a negative nitrogen balance.

The nurse is explaining to the client that at rest, the body uses energy that is included in the basal metabolic rate (BMR). What is the BMR? The caloric cost of day-to-day activities The caloric cost of staying alive The caloric cost of mild activity The caloric cost of studying

The caloric cost of staying alive

A 71-year-old female client has functional limitations, in part caused by obesity. For which of the following reasons would the client seem reluctant to discussing a weight loss plan with the nurse? The client feels that losing weight can cause a strain on the body. The client feels that she would feel worse if she lost weight. The client does not feel that her weight impedes her functionality. The client does not feel the need to make changes at this point in her life.

The client does not feel the need to make changes at this point in her life.

While taking a client's medical psychosocial history, the nurse collects client data which includes the following information: The client has been treated for a major depressive episode 2 years ago, the client practices Islam, and is high in socioeconomic status. Her hair is visibly dry and dull, and she mentions that her hands feel weak in the past few months. What in the medical-psychosocial assessment might lead to a nutritional deficiency? 1. The client is likely fasting for a religious ritual. 2. The client's culture and religion 3. The client's hair is dry and dull. 4. The client's history of major depressive episode

The client's history of major depressive episode

The nurse is aware that the production of pale yellow urine is assumed to be a reliable indicator of adequate fluid intake. However, in certain age groups thirst is only recognized after significant fluid loss. For which of the following age groups is this true? Teenagers The obese The elderly Middle-aged adults

The elderly

Physiologically and developmentally, an infant will indicate when he or she is ready and able to have solid foods added to his or her diet. What indication in an infant would tell the nurse that he or she is ready to progress to solid foods? The eruption of teeth The ability to self-feed When the infant can sit with support When the infant can turn his or her head

The eruption of teeth

The nurse is teaching a class on a high-fiber diet. The class correctly recognizes that clients may require a fiber supplement for which of the following reasons? The fiber content of fruits, vegetables, and whole wheat bread is low. High-fiber foods are expensive and difficult to access. The best sources for high-fiber are only available seasonally. Most people do not consume enough fruits and vegetables

The fiber content of fruits, vegetables, and whole wheat bread is low.

The nurse is teaching a class on a high-fiber diet. The class correctly recognizes that clients may require a fiber supplement for which of the following reasons? The fiber content of fruits, vegetables, and whole wheat bread is low. Most people do not consume enough fruits and vegetables. The best sources for high-fiber are only available seasonally. High-fiber foods are expensive and difficult to access.

The fiber content of fruits, vegetables, and whole wheat bread is low.

The nurse is talking with the nutrition class about how the normal response to severe acute stress impacts metabolism. One of the students asks what the intensity of stress response is based on. What is the best response? The intensity of the stress response depends on the gender and previous physical health of the client. The intensity of the stress response depends on the cause and the severity of the initial injury. The intensity of the stress response depends on the age and previous nutritional health of the client. The intensity of the stress response depends on the age and gender of the client.

The intensity of the stress response depends on the cause and the severity of the initial injury.

The nurse has been asked to speak on diet and nutrition at a local community center. During the presentation, the nurse discusses the importance of "Adequate Intake." Which of the following should the nurse be sure to include regarding this topic? 1. The primary purpose of the Adequate Intake is to have a way to compare nutritional patterns to the Recommended Daily Allowances (RDA). 2. The primary purpose of the Adequate Intake is to help people calculate an estimate of daily nutritional intake. 3. The primary purpose of the Adequate Intake is to provide a nutritional goal for people to try to achieve. 4. The primary purpose of the Adequate Intake is to provide the optimal level of intake for all nutrients.

The primary purpose of the Adequate Intake is to provide a nutritional goal for people to try to achieve.

The nurse should recommend which of the following when discussing adequate fiber intake for a client with diabetes? Most fiber in the form of insoluble fiber More fiber than currently recommended for the general population The same amount of fiber as currently recommended for the general population Less fiber than currently recommended for the general population

The same amount of fiber as currently recommended for the general population

The nurse is discussing intake of fat with a client who has recently been diagnosed with type 2 diabetes. Which of the following information is true about fat intake for people with diabetes? Saturated fats may help with effective glucose regulation. Eating foods like margarine may increase the risk of cardiovascular disease. The type of fat is more important than the total amount. Daily fat intake should be limited to less than 20 g/day.

The type of fat is more important than the total amount.

The nurse is aware that magnesium deficiency is a potential nutritional problem in the United States. What is the cause of this potential problem? Not drinking enough enriched orange juice Not eating enough citrus fruits in our diet Low intake of milk and dairy products The use of refined grains over whole grains

The use of refined grains over whole grains

The nurse is discussing the role of micronutrients in the regulation of blood glucose in diabetes. Which of the following statements is true? Zinc can support the regulation of insulin. There is insufficient evidence to support the use of micronutrient supplements. Manganese can help with glucose metabolism. Potassium can help with blood glucose regulation.

There is insufficient evidence to support the use of micronutrient supplements.

A nurse is discussing dietary minerals with a client who wants to ensure as much nutritional value of food is retained during food preparation. The nurse should provide which of the following information about minerals? They are released from foods only by cooking. They are made dormant by heat and light. They are oxidized by cooking. They are not destroyed by heat and light.

They are not destroyed by heat and light.

Parental support for a healthier lifestyle is vital to children. Parents also recognize that setting a good example for their children is important. Why do some parents who are overweight feel they cannot set a good example in diet and nutrition for their children? They feel they cannot practice what they preach. They feel their children eat fast food too often. They feel their children are not motivated. They feel their children do not listen to them.

They feel they cannot practice what they preach.

The nurse is caring for a client in a long-term care facility who is 77 years of age and overweight. The nurse is considering putting this client on a low-calorie diet. Some experts recommend that strict therapeutic diets be used in older adults only when a significant improvement in health can be expected. Which elderly clients would a restrictive diet be recommended for? Those with high cholesterol levels Those with high potassium levels Those with mild hypertension Those with liver cirrhosis

Those with liver cirrhosis

The nurse is admitting a 35-year-old client with alcoholism to the hospital. In planning nursing care, what long-term nutritional goals might the nurse set? 1. To replenish fluid losses 2. To alleviate side effects of treatments 3. To improve eating habits to reduce the risk of chronic disease 4. To alleviate symptoms of a disease

To improve eating habits to reduce the risk of chronic disease

The nurse explains to the family members of a client with acute respiratory distress syndrome (ARDS) that nutritional support is required for which of the following reasons? To increase empty-calorie liquids To increase calorie and protein intake To limit fiber To limit total fluid intake

To increase calorie and protein intake

The nurse is asked to discuss weight loss drugs to a group of bariatric clients. Which of the following information should the nurse include regarding phentermine/topiramate? It is approved for approximately 6 months of use. It works by making the body think it is full. Tolerance may develop after only a few weeks. It is a peripheral nervous system drug.

Tolerance may develop after only a few weeks.

When providing client teaching regarding what causes the diarrhea that accompanies Crohn disease, which of the following is the nurse's best response? 1. The bowels draw in too much water making the stools loose. Consume fluids over the daily requirements. Antibiotics have destroyed the natural bacteria in the bowel. Too much fluid and electrolytes make their way into the bowel.

Too much fluid and electrolytes make their way into the bowel.

When teaching a nutrition class, the nurse educator discusses the importance of trace minerals in body function. A lack of information means the nurse educator can only provide basic information about trace minerals for which of the following reasons? Trace minerals are difficult to study; bioavailability varies within the context of the total diet. When trace minerals have been studied; the comparability of trace mineral content of foods is high. Data about trace minerals is available; reliable and valid indicators of trace element status can be found in research. Information about trace minerals is difficult to obtain; food composition data are not required for all trace minerals.

Trace minerals are difficult to study; bioavailability varies within the context of the total diet.

The nurse is admitting a client onto the unit with a diagnosis of manganese toxicity. What symptoms would be expected in this client? Tremors Tetany Flexibility Unaltered gait

Tremors

The mother of a 7-month-old infant tells the nurse she has been putting fruit juice in a bottle for her baby to drink. Which of the following is the nurse's best response? Fresh squeezed juice can replace a meal. Juice should never be given to an infant. Try offering the fruit juice in a cup. Alternate juice with milk.

Try offering the fruit juice in a cup.

The diet is influenced by all foods and drinks consumed, which includes alcohol. What is the current recommendation for alcohol consumption according to the various published guidelines? 1. Two drinks each day for men, one drink each day for women 2. One drink each day for men and women 3. Two drinks each day for men and women 4. One drink each day for men, two drinks each day for women

Two drinks each day for men, one drink each day for women

Part of the nutrition assessment is calculating the client's BMI. The nurse is aware that a BMI of 18 indicates the client falls in what category? 1. Obese 2. Underweight 3. Healthy weight 4. Overweight

Underweight

The nurse is preparing to discharge a client who has been treated for fecal impaction due to chronic constipation. Which of the following foods can the nurse recommend that will act as a laxative? Substituting beans for meat All dried fruit for snacks All whole wheat products Unprocessed wheat bran

Unprocessed wheat bran

The nurse is working with a client who has had difficulty maintaining adequate control of his type 2 diabetes. During the course of the conversation, the client mentions that he gets tired of adding up all the calories and forgets to do it sometimes. What suggestion can the nurse make to help this client? Download an app to his smartphone from the ADA to help count calories. Make a daily plan and follow it precisely. Use a dinner plate to plan meals. Concentrate on checking his glucose before each meal.

Use a dinner plate to plan meals.

A client tells the nurse he is having difficulty finding a practical way to ensure he remains within the daily calorie limit that will promote weight loss. Which of the following tips will most likely help the client? Opt for low-fat foods. Use a smaller plate for meals. Eat fruit in place of vegetables. Eat most of your calories at breakfast.

Use a smaller plate for meals.

The nurse is counseling a client recovering from a major burn trauma. She is about to be released to go home and finish her recovery. The physician has ordered a high-protein diet for her at home, and the nurse is teaching her methods of increasing protein density in her diet. What would be the best method for this client to increase the protein density in her diet? Use honey on toast, cereal, and in coffee. Use double-strength milk (liquid milk fortified with skim milk powder). Add butter to hot foods such as potatoes, cooked cereals, and soups. Substitute mayonnaise for salad dressing.

Use double-strength milk (liquid milk fortified with skim milk powder).

The nurse is meeting with a client who currently has no health concerns and has a history of unsuccessful restrictive dieting for being overweight. The nurse knows the most effective way for the client to control weight and sustain good health is which of the following? Counting daily intake of fat Removing carbohydrates from the diet Using a nondiet approach to weight control Bariatric surgical intervention

Using a nondiet approach to weight control

When planning treatment with a client with obesity, it is important for the nurse to include which of the following behavior modification techniques? Using a small plate Never eating forbidden foods Skipping meals Eating rapidly

Using a small plate

The nurse is assessing a new client who is being admitted due to possible mineral toxicity. When taking the client's nutritional history, the nurse is most likely to find that the client has been doing which of the following? Having mineral interactions from food intake Has been consuming home-processed foods Using excessive mineral supplements Making unbalanced choice of foods

Using excessive mineral supplements

When meal planning, a client with type 2 diabetes asks the nurse which foods can be eaten without carbohydrate counting. Which of the following food should the nurse suggest? Milk Potatoes Bananas Vegetable broth

Vegetable broth

The risk of nutritional and health problems is present in all age groups. Which of the following is the most likely nutritional risk during adolescence? Protein deficiency Sodium deficiency Vitamin A deficiency Vitamin D deficiency

Vitamin A deficiency

The nurse is providing education on the prevention of major fractures in older adults, such as fracture of the hip. Which of the following recommendations for vitamin intake should the nurse be sure to include in the educational session? Vitamin E Vitamin B6 Vitamin B12 Vitamin D

Vitamin D

Which of the following supplements should the nurse emphasize are needed in the older adult population? Vitamin B6 Vitamin K Niacin Vitamin D

Vitamin D

The nurse is helping a client try to understand why the family is now putting on weight after a recent move from another country. The nurse suggests which of the following may be contributing to this problem? Weight gain is determined by family genetics. Weight gain is dependent on an obesogenic environment. Weight gain is short term and related to the recent move. Weight gain is not concerning unless it becomes obesity.

Weight gain is dependent on an obesogenic environment.

The nurse is a member of a panel discussing the optimal type of weight loss diet. When the panel members ask the audience what the most important thing to remember about weight loss is, which of the following responses indicates the discussion was effective? Weight loss is dependent on the amount of the calories consumed. Weight loss is dependent on the nutritional content of the calories consumed. Weight loss depends on the meal at which the calories are consumed. Weight loss is dependent on the source of the calories consumed.

Weight loss is dependent on the amount of the calories consumed.

The nurse is assisting a client understand the gluten-free diet. Teaching has been effective when the client identifies which of the following foods should be avoided? Wheat, rye, and barley Wheat, oats, and rice Corn, oats, and rice Oats, corn, and barley

Wheat, rye, and barley

Drugs are available to aid in weight loss, but they are not for everyone. When is a person a candidate for drug therapy in the treatment of obesity? When the person has not been able to lose weight any other way When a man's waist is 35 in. or more When the person can maintain a 1500-calorie diet for at least 1 month When a BMI is 30 or greater

When a BMI is 30 or greater

The family members of an older adult client in a long-term care facility are concerned that a restrictive diet will decrease quality of life for the client. The nurse correctly explains that the diet must be adhered to for which of the following reasons? When it is for the purpose of hospice care When a significant improvement in health can be expected When it is intermittent as a form of detoxification When a resident has decreased intake and weight loss

When a significant improvement in health can be expected

To ensure all inpatients receive adequate care, The Joint Commission specifies that nutrition screening be conducted at what time during hospital admission? 1. Within 24 hours after admission 2. Within 36 hours after admission 3. Within 12 hours after admission 4. Immediately upon admission

Within 24 hours after admission

When providing teaching to a client newly diagnosed with type 2 diabetes, what should the nurse tell the client about nutrient recommendations for the intake of fat? Polyunsaturated fat should provide up to 20% of total calories. Ensure trans fat consumption is equal to monounsaturated fat. The level of monounsaturated fat should be the total daily calorie intake. Worry about the type of fat, not so much the amount.

Worry about the type of fat, not so much the amount.

The nurse is conducting client education with a client recovering from major trauma and his wife before he is discharged from the hospital. A high-calorie, high-protein diet is being recommended to optimize the client's recovery. The nurse should suggest which of the following as a high-protein snack? Italian ice Granola Yogurt Cream cheese on toast

Yogurt

The nurse educator is teaching a nutrition class for nursing students. The discussion for today is about the nutrition therapy for dumping syndrome. The nurse educator should include which of the following interventions manage symptoms of this health complication? Select all that apply. a. Only one or two items are eaten at each meal. b. Fat is eliminated from the diet. c. Small, frequent feedings are the first feedings. d. Complex carbohydrates are a part of the diet. e. Clear liquids are the first feeding.

a. Only one or two items are eaten at each meal. c. Small, frequent feedings are the first feedings. d. Complex carbohydrates are a part of the diet.

The nurse is conducting during the discharge of a client recovering from an exacerbation of peptic ulcer disease. What statement by the client lets the nurse know that the client understands the teaching? a. "I should season my food well with the spices I like best." b. "I should not eat for 2 hours before going to bed." c. "Drinking caffeine throughout the day is recommended." d. "I should eat what I like when I like."

b. "I should not eat for 2 hours before going to bed."

For a client with gastroesophageal reflux disease (GERD), the nurse should recommend which of the following foods should be eliminated from the diet? Select all that apply. a. Mashed potatoes b. Cola c. Chocolate ice cream d. Orange juice e. Peppermint candy

b. Cola c. Chocolate ice cream e. Peppermint candy

The nurse is discussing anorexia in clients with depression with the nutrition class. Which of the following interventions should the nurse discuss when treating clients with this problem? a. Take antidepressant medications with food. b. Create a meal plate that is visually appealing. c. Add a favorite drink to each meal. d. Encourage the client to eat fried food

b. Create a meal plate that is visually appealing.

The nurse is assisting a client with dysphagia who is prescribed honey-like liquids to plan a daily menu. Which of the following liquids would be appropriate for this client? a. Cream soup b. Instant cocoa c. Yogurt d. Popsicle

c. Yogurt

While providing counseling to a client with depression, the nurse notes the client has had a significantly decreased appetite. What should the nurse suggest to this client that might help improve her intake? a. Increase fat content of each meal and snack. b. Have one larger meal to replace two to three meals. c. Have meals alone in an area where distractions are limited. d. Avoid fluids 30 minutes before and after a meal.

d. Avoid fluids 30 minutes before and after a meal.

The nurse is providing teaching to a client regarding the prevention of dumping syndrome after having bariatric surgery. Which of the following foods should the nurse recommend as safe to consume for this client? a. Shredded wheat b. Apples c. Kidney beans d. Cream of wheat

d. Cream of wheat

A client is being discharged from hospital after recovering from a head trauma. Ongoing pain is causing the client to experience anorexia. What should the nurse include in the client's discharge teaching? a. Give pain medications with each meal. b. Give small meals. c. Schedule pain medications when they do not interfere with meals. d. Give pain medications prior to meals.

d. Give pain medications prior to meals.


Set pelajaran terkait

Life & Health Insurance Exam Review

View Set

Working and Communicating with Families

View Set

Abnormal Psychology Chapter 4- Clinical Assessment and Diagnosis Study Review

View Set

10: Making Capital Investment Decisions

View Set